Термодинамика физика формулы: Ошибка: 404 Материал не найден

Содержание

основные формулы и законы. Первый закон термодинамики, КПД

Основные формулы термодинамики и молекулярной физики, которые вам пригодятся. Еще один отличный день для практических занятий по физике. Сегодня мы соберем вместе формулы, которые чаще всего используются при решении задач в термодинамике и молекулярной физике.

Ежедневная рассылка с полезной информацией для студентов всех направлений – на нашем телеграм-канале.

Итак, поехали. Попытаемся изложить законы и формулы термодинамики кратко.

Идеальный газ

Идеальный газ – это идеализация, как и материальная точка. Молекулы такого газа являются материальными точками, а соударения молекул – абсолютно упругие. Взаимодействием же молекул на расстоянии пренебрегаем. В задачах по термодинамике реальные газы часто принимаются за идеальные. Так гораздо легче жить, и не нужно иметь дела с массой новых членов в уравнениях.

Итак, что происходит с молекулами идеального газа? Да, они движутся! И резонно спросить, с какой скоростью? Конечно, помимо скорости молекул нас интересует еще и общее состояние нашего газа. Какое давление P он оказывает на стенки сосуда, какой объем V занимает, какая у него температура T.

Для того, чтобы узнать все это, есть уравнение состояния идеального газа, или уравнение Клапейрона-Менделеева

Здесь m – масса газа, M – его молекулярная масса (находим по таблице Менделеева), R – универсальная газовая постоянная, равная 8,3144598(48) Дж/(моль*кг).

Универсальная газовая постоянная может быть выражена через другие константы (постоянная Больцмана и число Авогадро

)

Массу, в свою очередь, можно вычислить, как произведение плотности и объема.

Основное уравнение молекулярно-кинетической теории (МКТ)

Как мы уже говорили, молекулы газа движутся, причем, чем выше температура – тем быстрее. Существует связь между давлением газа и средней кинетической энергией E его частиц. Эта связь называется основным уравнением молекулярно-кинетической теории и имеет вид:

Здесь n – концентрация молекул (отношение их количества к объему), E – средняя кинетическая энергия. Найти их, а также среднюю квадратичную скорость молекул можно, соответственно, по формулам:

Подставим энергию в первое уравнение, и получим еще один вид основного уравнения МКТ

Первое начало термодинамики. Формулы для изопроцессов

Напомним Вам, что первый закон термодинамики гласит: количество теплоты, переданное газу, идёт на изменение внутренней энергии газа U и на совершение газом работы A. Формула первого закона термодинамики записывается так:

Как известно, с газом что-то происходит, мы можем сжать его, можем нагреть. В данном случае нас интересуют такие процессы, которые протекают при одном постоянном параметре. Рассмотрим, как выглядит первое начало термодинамики в каждом из них.

Кстати! Для всех наших читателей сейчас действует скидка 10% на любой вид работы

Изотермический процесс протекает при постоянной температуре. Тут работает закон Бойля-Мариотта: в изотермическом процессе давление газа обратно пропорционально его объёму. В изотермическом процессе:

Изохорный процесс протекает при постоянном объеме. Для этого процесса характерен закон Шарля: При постоянном объеме давление прямо пропорционально температуре. В изохорном процессе все тепло, подведенное к газу, идет на изменение его внутренней энергии.

Изобарный процесс идет при постоянном давлении. Закон Гей-Люссака гласит, что при постоянном давлении газа его объём прямо пропорционален температуре. При изобарном процессе тепло идет как на изменение внутренней энергии, так и на совершение газом работы.

Адиабатный процесс. Адиабатный процесс – это такой процесс, который проходит без теплообмена с окружающей средой. Это значит, что формула первого закона термодинамики для адиабатного процесса выглядит так:

Внутренняя энергия одноатомного и двухатомного идеального газа

Теплоемкость

Удельная теплоемкость равна количеству теплоты, которое необходимо для нагревания одного килограмма вещества на один градус Цельсия.

Помимо удельной теплоемкости, есть молярная теплоемкость (количество теплоты, необходимое для нагревания одного моля вещества на один градус) при постоянном объеме, и молярная теплоемкость при постоянном давлении. В формулах ниже, i – число степеней свободы молекул газа. Для одноатомного газа i=3, для двухатомного – 5.

Тепловые машины. Формула КПД в термодинамике

Тепловая машина, в простейшем случае, состоит из нагревателя, холодильника и рабочего тела. Нагреватель сообщает тепло рабочему телу, оно совершает работу, затем охлаждается холодильником, и все повторяется вновь. Типичным примером тепловой машины является двигатель внутреннего сгорания.

Коэффициент полезного действия тепловой машины вычисляется по формуле

Вот мы и собрали основные формулы термодинамики, которые пригодятся в решении задач. Конечно, это не все все формулы из темы термодинамика, но их знание действительно может сослужить хорошую службу. А если возникнут вопросы  – помните о студенческом сервисе, специалисты которого готовы в любой момент прийти на выручку.

основные формулы и законы. Первый закон термодинамики, КПД

Основные формулы термодинамики и молекулярной физики, которые вам пригодятся. Еще один отличный день для практических занятий по физике. Сегодня мы соберем вместе формулы, которые чаще всего используются при решении задач в термодинамике и молекулярной физике.

Ежедневная рассылка с полезной информацией для студентов всех направлений – на нашем телеграм-канале.

Итак, поехали. Попытаемся изложить законы и формулы термодинамики кратко.

Идеальный газ

Идеальный газ – это идеализация, как и материальная точка. Молекулы такого газа являются материальными точками, а соударения молекул – абсолютно упругие. Взаимодействием же молекул на расстоянии пренебрегаем. В задачах по термодинамике реальные газы часто принимаются за идеальные. Так гораздо легче жить, и не нужно иметь дела с массой новых членов в уравнениях.

Итак, что происходит с молекулами идеального газа? Да, они движутся! И резонно спросить, с какой скоростью? Конечно, помимо скорости молекул нас интересует еще и общее состояние нашего газа. Какое давление P он оказывает на стенки сосуда, какой объем V занимает, какая у него температура T.

Для того, чтобы узнать все это, есть уравнение состояния идеального газа, или уравнение Клапейрона-Менделеева

Здесь m – масса газа, M – его молекулярная масса (находим по таблице Менделеева), R – универсальная газовая постоянная, равная 8,3144598(48) Дж/(моль*кг).

Универсальная газовая постоянная может быть выражена через другие константы (постоянная Больцмана и число Авогадро)

Массу, в свою очередь, можно вычислить, как произведение плотности и объема.

Основное уравнение молекулярно-кинетической теории (МКТ)

Как мы уже говорили, молекулы газа движутся, причем, чем выше температура – тем быстрее. Существует связь между давлением газа и средней кинетической энергией E его частиц. Эта связь называется основным уравнением молекулярно-кинетической теории и имеет вид:

Здесь n – концентрация молекул (отношение их количества к объему),

E – средняя кинетическая энергия. Найти их, а также среднюю квадратичную скорость молекул можно, соответственно, по формулам:

Подставим энергию в первое уравнение, и получим еще один вид основного уравнения МКТ

Первое начало термодинамики. Формулы для изопроцессов

Напомним Вам, что первый закон термодинамики гласит: количество теплоты, переданное газу, идёт на изменение внутренней энергии газа U и на совершение газом работы A. Формула первого закона термодинамики записывается так:

Как известно, с газом что-то происходит, мы можем сжать его, можем нагреть. В данном случае нас интересуют такие процессы, которые протекают при одном постоянном параметре. Рассмотрим, как выглядит первое начало термодинамики в каждом из них.

Кстати! Для всех наших читателей сейчас действует скидка 10% на любой вид работы

Изотермический процесс протекает при постоянной температуре. Тут работает закон Бойля-Мариотта: в изотермическом процессе давление газа обратно пропорционально его объёму. В изотермическом процессе:

Изохорный процесс протекает при постоянном объеме. Для этого процесса характерен закон Шарля: При постоянном объеме давление прямо пропорционально температуре. В изохорном процессе все тепло, подведенное к газу, идет на изменение его внутренней энергии.

Изобарный процесс идет при постоянном давлении. Закон Гей-Люссака гласит, что при постоянном давлении газа его объём прямо пропорционален температуре. При изобарном процессе тепло идет как на изменение внутренней энергии, так и на совершение газом работы.

Адиабатный процесс. Адиабатный процесс – это такой процесс, который проходит без теплообмена с окружающей средой. Это значит, что формула первого закона термодинамики для адиабатного процесса выглядит так:

Внутренняя энергия одноатомного и двухатомного идеального газа

Теплоемкость

Удельная теплоемкость равна количеству теплоты, которое необходимо для нагревания одного килограмма вещества на один градус Цельсия.

Помимо удельной теплоемкости, есть молярная теплоемкость (количество теплоты, необходимое для нагревания одного моля вещества на один градус) при постоянном объеме, и молярная теплоемкость при постоянном давлении. В формулах ниже, i – число степеней свободы молекул газа. Для одноатомного газа i=3, для двухатомного – 5.

Тепловые машины. Формула КПД в термодинамике

Тепловая машина, в простейшем случае, состоит из нагревателя, холодильника и рабочего тела. Нагреватель сообщает тепло рабочему телу, оно совершает работу, затем охлаждается холодильником, и все повторяется вновь. Типичным примером тепловой машины является двигатель внутреннего сгорания.

Коэффициент полезного действия тепловой машины вычисляется по формуле

Вот мы и собрали основные формулы термодинамики, которые пригодятся в решении задач. Конечно, это не все все формулы из темы термодинамика, но их знание действительно может сослужить хорошую службу. А если возникнут вопросы  – помните о студенческом сервисе, специалисты которого готовы в любой момент прийти на выручку.

Основные формулы термодинамики

В термодинамике изучают самые общие законы и физические процессы преобразований внутренней энергии. При этом считается, что любое материальное тело имеет тепловую энергию $U$, которая зависит от его температур.

Перед тем, как рассмотреть основные термодинамические формулы необходимо дать определение термодинамике.

Определение 1

Термодинамика – это обширный раздел физики, который исследует и описывает процессы, происходящие в системах, а также их состояния.

Указанное научное направление опирается на обобщенные факты, которые были получены опытным путем. Происходящие в термодинамических концепциях явления описываются посредством использования макроскопических величин.

В их список входят такие параметры, как:

  • давление;
  • температура;
  • концентрация;
  • энергия;
  • объем.

К отдельным молекулам данные параметры неприменимы, а сводятся к детальному описанию системы в общем ее виде. Много решений, которые основаны на термодинамических законах, можно встретить в сфере электроэнергетики и тепловой техники. Что и свидетельствует о понимании фазовых переходов, химических процессов и явлений переноса. В некотором роде термодинамика тесно “сотрудничает” с квантовой динамикой.

Уравнение идеального газа в термодинамике

Рисунок 1. Работа в термодинамике. Автор24 — интернет-биржа студенческих работ

Определение 2

Идеальный газ – это некая идеализация, такая же, как и материальная точка.

Готовые работы на аналогичную тему

Молекулы такого элемента являются материальными точками, а соударения частиц – абсолютно упругие и постоянные. В задачах по термодинамике реальные газы зачастую принимаются за идеальные. Так гораздо легче составлять формулы, и не нужно иметь дела с огромным количеством новых величин в уравнениях.

Итак, молекулы идеального газа движутся, а вот чтобы узнать с какой скоростью и массой, необходимо использовать уравнение состояния идеального газа, или формулу Клапейрона-Менделеева: $PV = \frac{m}{M}RT$. Здесь $m$ – масса исследуемого газа, $M$ – его изначальная молекулярная масса, $R$ – универсальная постоянная, равная 8,3144598 Дж/(моль*кг).

В этом аспекте массу идеального газа также можно вычислить, как произведение объема и плотности $m = pV$. Существует некая связь между средней кинетической энергией $E$ и давлением газа. Эта взаимосвязь называется в физике основным уравнением молекулярно-кинетической теории и имеет вид: $p = \frac{2}{3}nE$, где $n$ – концентрация движущихся молекул по отношению к общему объему, $E$ – коэффициент средней кинетической энергии.

Первое начало термодинамики. Формулы для изопроцессов

Рисунок 2. Уравнение состояния идеального газа. Автор24 — интернет-биржа студенческих работ

Первый термодинамический закон гласит: количество внутренней теплоты, переданное газу, идёт только на изменение общей энергии газа $U$ и на совершение веществом работы $A$. Формула первого начала термодинамики записывается так: $Q = ΔU + A$.

как известно, с газом в системе всегда что-то происходит, ведь его можно сжать или нагреть. В данном случае необходимо рассмотреть такие процессы, которые протекают при одном стабильном параметре. Первое начало термодинамики в изотермическом случае, который протекает при постоянной температуре, задействует закон Бойля-Мариотта.

В результате изотермического процесса давление газа обратно пропорционально его изначальному объёму: $Q = A.$

Изохорный – наблюдается при постоянном объеме. Для этого явление применим закон Шарля, согласно которому, давление прямо пропорционально общей температуре. В изохорном процессе все подведенное к газу тепло идет на изменение его внутренней энергии и записывается в таком виде: $Q = ΔA.$

Изобарный процесс – происходит при постоянном давлении. Закон Гей-Люссака предполагает, что при неизменном давлении идеального газа его начальный объём прямо пропорционален итоговой температуре. При изобарном процессе тепло идет на совершение газом работы и на изменение внутреннего энергетического потенциала: $Q = \Delta U+p\Delta V.$

Формула теплоемкости и главная формула КПД в термодинамике

Рисунок 3. Количество теплоты. Автор24 — интернет-биржа студенческих работ

Замечание 1

Удельная теплоемкость в термодинамической системе всегда равна количеству теплоты, которое выделяется для нагревания одного килограмма действующего вещества на один градус Цельсия.

Уравнение теплоемкости записывается таким образом: $c = \frac{Q}{m\Delta t}$. Помимо указанного параметра, существует и молярная теплоемкость, которая работает при постоянном объеме и давлении.

Ее действия видно в следующей формуле: $C_v = \frac {i}{2}R$ где $i$ – количество степеней свободы молекул газа.

Тепловая машина, в самом простейшем случае, состоит из холодильника, нагревателя и рабочего материального тела. Нагреватель изначально сообщает тепло физическому веществу и совершает определенную работу, а затем постепенно охлаждается холодильником, и все повторяется по кругу. Типичным примером тепловой машины выступает двигатель внутреннего сгорания.

Коэффициент полезного действия теплового устройства вычисляется по формуле: $n = \frac {Q_h-Q_x }{Q_h }.$

При изучении основ и уравнений термодинамики следует понять, что на сегодняшний день существует два метода описания физических процессов, происходящих в макроскопических материальных телах: статистический и термодинамический.

Методы термодинамики и ее формулы позволяет раскрыть и описать смысл экспериментальных закономерностей в виде закона Менделеева-Клапейрона. Важно понять, что в термодинамических концепциях, в отличие от систем молекулярной физики, не изучаются конкретные взаимодействия, происходящие с определенными молекулами или атомами, а рассматривается постоянные взаимопревращения и связь разнообразных видов теплоты, энергии и работы.

Уравнение состояния и его функции

Рисунок 4. Термодинамические уравнения состояния. Автор24 — интернет-биржа студенческих работ

При исследовании макросостояний применяются функции состояния, которые предполагают показатель, демонстрирующий определённые состояния термодинамического равновесия, независящий от предыстории концепции и метода её перехода в абсолютное состояние.

Основными функциями состояния при грамотном построении термодинамики являются:

  • внутренняя энергия;
  • энтропия;
  • температура;
  • термодинамические потенциалы.

Однако функции состояния в термодинамики не являются полностью независимыми, и для однородной системы любой термодинамический принцип может быть записан как выражение двух самостоятельных переменных. Такие функциональные взаимосвязи называются уравнениями общего состояния.

На сегодняшний день различают такие виды уравнений:

  • термическое уравнение состояние – определяющее связь между давлением, температурой и объёмом;
  • калорическое уравнение – выражающее внутренний энергетический потенциал, как функцию от объёма и температуры;
  • каноническое уравнение состояние – записываемое в качестве термодинамического потенциала в соответствующих переменных.

Знание уравнения состояния очень важно для использования на практике общих принципов термодинамики. Для каждой конкретной термодинамической концепции такие выражения определяются из опыта или способами статистической механики, и в пределах термодинамики оно считается заданным при изначальном определении системы.

Формулы термодинамики

Определение и формулы термодинамики

При этом считается, что любое тело имеет внутреннюю энергию (U), которая зависит от его температуры:

   

где i – число степеней свободы молекулы; m – масса; – молярная масса; – универсальная газовая постоянная; T – температура по абсолютной шкале.

При теплообмене количество теплоты (Q) служит мерой изменения внутренней энергии. Количество теплоты, которое получает тело массы m при увеличении его температуры на величину равную , равно:

   

где – удельная теплоемкость вещества. В общем случае теплоемкость тела (C) определена как:

   

В соответствии с первым началом термодинамики, теплота, которую получает термодинамическая система (), расходуется ей на совершение работы (A) и изменение ее внутренней энергии ():

   

Для элементарного изменения состояния термодинамической системы первый закон термодинамики записывают как:

   

или:

   

где p – давление; – элементарное изменение объема.

Термодинамическим коэффициентом полезного действия (КПД) () называют отношение работы (A), которое совершает рабочее тело к количеству теплоты (), которое получает данное тело:

   

где — количество теплоты, отданное рабочим телом холодильнику.

Для цикла Карно, который состоит из двух изотерм и двух адиабат и проводится с идеальным газом КПД равно:

   

где – температура нагревателя; – температура холодильника.

Энтропия в термодинамике

Энтропией называют функцию состояния термодинамической системы, элемент которой в обратимом процессе равен:

   

В соответствии со вторым началом термодинамики в необратимом элементарном процессе изменение энтропии:

   

Для адиабатного процесса выражение (10) имеет вид:

   

где знак равно относится к обратимому процессу. Выражение (11) – математическая запись второго начала термодинамики (Следует помнить, что рассматривается замкнутая система).

Работа в термодинамике вычисляется как:

   

где – начальный объем системы; – конечный объем. Работа считается большей нуля, если работу выполняет система (газ) над внешними силами.

Примеры решения задач по теме «Термодинамика»

Основные формулы и методические рекомендации по решению задач на основы термодинамики

Данная тема будет посвящена повторению основ термодинамики. Также будут рассмотрены некоторые формулы, относящиеся к данному разделу физики, и даны некоторые общие рекомендации по решению задач на данную тему.

Под термодинамической системой подразумевается любое конечных размеров макротело или совокупность макротел.

Под макротелом имеется ввиду тело, размеры которого велики по сравнению с атомными размерами и которое состоит из огромного числа микрочастиц.

В отличии от состояния механической системы, определяемого совокупностью координат и скоростей тел, входящих в нее, состояние термодинамической системы определяется набором значений термодинамических параметров, то есть физических величин, характеризующих свойства системы в целом.

Термодинамические параметры являются макровеличинами, то есть такими величинами, которые могут быть либо непосредственно измерены с помощью приборов, либо выражены через другие, непосредственно измеряемые на опыте, величины.

Состояние простейших термодинамических систем (например, однородных газов и жидкостей, не подверженных действию каких-либо полей) определяется давлением, объемом и температурой.

Известно, что согласно молекулярно-кинетической теории все вещества состоят из частиц, которые находятся в непрерывном тепловом движении и взаимодействуют друг с другом. Поэтому, даже если тело неподвижно и имеет нулевую потенциальную энергию, оно обладает энергией — внутренней энергией.

Под внутренней энергией в термодинамике понимают сумму кинетической энергии поступательного движения молекул и потенциальной энергии их взаимодействия.

    

Внутренняя энергия тела определяет его тепловое состояние и изменяется при переходе из одного состояния в другое. В данном состоянии тело обладает вполне определенной внутренней энергией, не зависящей от того, в результате какого процесса оно перешло в данное состояние. Рассчитать внутреннюю энергию можно только для идеального газа.

Из формулы видно, что внутренняя энергия идеального газа зависит только от температуры и числа молекул и не зависит ни от объема, ни от давления. Поэтому изменение внутренней энергии идеального газа определяется только изменением его температуры.

Таким образом, внутренняя энергия тела может изменяться двумя способами: либо при совершении механической работы, либо в результате теплообмена.

К первому способу относят:

Действие сил трения, поскольку, как известно из опыта, трение всегда сопровождается изменением температуры трущихся тел.

Неупругое соударение двух и более тел. При таком соударении тел их кинетическая энергия уменьшается, она превращается во внутреннюю (например, если ударить несколько раз молотком по проволоке, лежащей на наковальне, — проволока нагреется).

А также деформацию. Ведь при деформации тел изменяются расстояния между частицами, из которых оно состоит, а, следовательно, изменяется потенциальная энергия их взаимодействия.

Второй способ изменения внутренней энергии — это теплопередача (или теплообмен) — происходит без совершения работы. Существует три вида теплообмена:

Теплопроводность — это процесс теплообмена между телами при их непосредственном контакте.

Конвекция — это теплопередача нагретыми потоками жидкости или газа от одних участков занимаемого ими объема в другие.

И излучение — это теплообмен, происходящий на расстоянии посредством электромагнитных волн.

При рассмотрении термодинамических процессов механическое перемещение макротел в целом не рассматривается. Поэтому понятие работы в термодинамике связывается с изменением объема тела, то есть с перемещением частей макротела друг относительно друга.

Важно запомнить, что сила давления газа совершает работу только в процессе изменения объема газа.

Что касается геометрического истолкования работы в термодинамике, то в случае изобарного процесса, в координатных осях p–V, работа равна площади заштрихованного прямоугольника.

Если же процесс не изобарный, то график процесса разбивают на большое количество изохор и изобар. Работа на изохорных участках равна нулю и работу газа находят как сумму работ на все изобарных участках.

Рассмотрим два основных закона термодинамики.

Начнем с первого, который еще называют первым началом термодинамики. Он выражает закон сохранения и превращения энергии в применении к тепловым процессам.

В общем случае, если механическая энергия системы не изменяется, а система не замкнута и между ней и окружающей средой происходит теплообмен, то изменяется внутренняя энергия системы.

Записанное уравнение выражает математическую запись первого начала термодинамики, которое формулируется так: изменение внутренней энергии при переходе термодинамической системы из одного состояния в другое равно работе внешних сил и количеству теплоты, переданному термодинамической системе в процессе теплообмена.

Если вместо работы внешних сил ввести работу системы над внешними телами, то выражение примет вид:

Тогда первый закон термодинамики можно сформулировать так: количество теплоты, сообщенное термодинамической системе, идет на изменение ее внутренней энергии и на совершение работы системой против внешних сил.

Из первого закона термодинамики вытекает невозможность создания вечного двигателя первого рода, то есть такого двигателя, который совершал бы работу без затраты энергии извне.

Рассмотрим возможность применения первого начала термодинамики к изопроцессам.

Начнем с изотермического процесса, то есть процесса изменения состояния газа при постоянной температуре и неизменной массе. При таком процессе все подведенное к газу количество теплоты идет на совершение газом работы.

В случае изохорного процесса — процесса при постоянном объеме, газ работу не совершает и энергия, сообщаемая газу путем теплообмена, расходуется целиком на увеличение его внутренней энергии.

При изобарном процессе, то есть когда не изменяется давление газа, подведенное к газу количество теплоты частично идет на увеличение его внутренней энергии, а частично на работу, совершаемую газом в процессе его расширения.

В термодинамике рассматривается еще один процесс — адиабатный.

Адиабатный процесс — это процесс, происходящий без теплообмена системы с окружающей средой.

В таком процессе система может совершать работу над внешними телами только за счет убыли своей внутренней энергии.

Таким образом, первый закон термодинамики требует, чтобы количество теплоты, отданное одним телом, в точности равнялось количеству теплоты, которое получит другое. А вот вопрос о том, от какого тела, от горячего к холодному или наоборот, перейдет энергия, остается открытым.

Направленность реальных тепловых процессов определяется вторым законом термодинамики, который был установлен непосредственным обобщением опытных фактов. Это постулат.

И так, невозможно перевести тепло от более холодной системы к более горячей при отсутствии других одновременных изменений в обеих системах или окружающих телах.

Из второго начала термодинамики вытекает невозможность создания вечного двигателя второго рода, то есть двигателя, который бы совершал работу за счет охлаждения какого-либо одного тела.

Тепловой двигатель — это устройство, совершающее механическую работу за счет внутренней энергии топлива. Он состоит из трех основных частей: рабочего тела, нагревателя и холодильника.

Все тепловые двигатели обладают общим свойством — периодичностью действия (или цикличностью).

Баланс энергии за цикл можно получить на основе первого закона термодинамики. Рабочему телу передается путем теплообмена некоторое количество теплоты, и над ним совершается работа. Рабочее тело само совершает работу при расширении и передает некоторое количество теплоты холодильнику.

Как и любой механизм, тепловой двигатель характеризуется коэффициентом полезного действия, который определяется отношением полезной работы, совершенной двигателем, к количеству теплоты, которое рабочее тело получило от нагревателя.

Из формул видно, что даже у идеальных тепловых двигателей коэффициент полезного действия меньше единицы.

Наибольший КПД, как показал впервые Сади Карно, можно получить для идеального теплового двигателя в случае, если рабочее тело совершает цикл Карно с тем же нагревателем и холодильником. Цикл Карно состоит из двух изотермических и двух адиабатных процессов.

Цикл Карно — это идеальный цикл. В реальных циклах нельзя осуществить идеальную адиабатность и изотермичность. КПД идеального теплового двигателя определяется по формуле

Сведем основные формулы термодинамики в таблицу.

Формула

Описание формулы

Внутренняя энергия идеального газа,  где i — число степеней свободы (i = 3 — для одноатомных молекул, i = 5 — для двух- и более атомных линейных молекул, i = 6 — для трех- и более атомных нелинейных молекул.

Количество теплоты, необходимое для нагревания тела, где с — удельная теплоемкость вещества, Т1 и Т2 — соответственно начальная и конечная температуры тела, m — масса тела.

Количество теплоты, необходимое для изменения агрегатного состояния вещества, где λ — удельная теплота плавления, L — удельная теплота парообразования.

Количество теплоты, выделяющееся при сгорании топлива, где q — удельная теплота сгорания топлива.

Первый закон термодинамики, где ΔU — изменение внутренней энергии системы, Авн — работа, произве-денная внешними силами над системой, А — работа, произведенная системой над внешними силами.

Уравнение теплового баланса.

Работа газа при изобарном процессе, где р — давление газа, ΔV — изменение объема газа.

КПД теплового двигателя, где А — полезная работа, совершаемая двигателем, Q1 — количество теплоты, полученное двигателем от нагревателя, Q2 — количество теплоты, отданное холодильнику.

Максимальное значение КПД теплового двигателя, где Т1 и Т2 — температуры нагревателя и холодильника соответственно.

 

Методические рекомендации по решению задач на данную тему.

1) Установить, какие тела входят в рассматриваемую термодинамическую систему.

2) Выяснить, что является причиной изменения внутренней энергии тел системы.

3) Если система адиабатически изолирована и замкнута, то необходимо установить, у каких тел системы внутренняя энергия увеличивается, а у каких уменьшается.

4) Составить уравнение теплового баланса (как частный случай первого закона термодинамики).

5) Если при взаимодействии двух тел внутренняя изменяется вследствие совершенной работы, то нужно установить, у какого из двух взаимодействующих тел изменяется внутренняя энергия и что является причиной этого изменения: работа, совершаемая самим телом, или работа, совершенная над телом.

6) Записать первый закон термодинамики.

7) При необходимости, дополните систему уравнением состояния или основным уравнением молекулярно-кинетической теории. Решите ее относительно искомой величины.

8) Если же при взаимодействии двух тел внутренняя изменяется вследствие совершенной работы, то нужно установить, у какого из двух взаимодействующих тел изменяется внутренняя энергия и что является причиной этого изменения: работа, совершаемая самим телом, или работа, совершенная над телом.

9) Если в задаче необходимо определить КПД цикла, то помните, что формула коэффициента полезного действия для цикла Карно выражает максимальный коэффициент полезного действия теплового двигателя.

Работа в термодинамике | Физика

Работа в механике и термодинамике. В механике работа определяется как произведение модулей силы и перемещения, умноженное на косинус угла между ними. Работа совершается при действии силы на движущееся тело и равна изменению кинетической энергии тела.

В термодинамике движение тела как целого не рассматривается и речь идет о перемещении частей макроскопического тела друг относительно друга. В результате меняется объем тела, а его скорость остается равной нулю. Следовательно, работа в термодинамике, определяемая так же, как и в механике, равна изменению не кинетической энергии тела, а его внутренней энергии.

Изменение внутренней энергии при совершении работы. Почему при сжатии или расширении меняется внутренняя энергия тела? Почему, в частности, нагревается воздух при накачивании велосипедной шины?

Причина изменения температуры в процессе сжатия газа состоит в следующем: при упругих соударениях молекул с движущимся поршнем их кинетическая энергия изменяется. При движении навстречу молекулам поршень передает им во время столкновений часть своей механической энергии, в результате чего газ нагревается. Поршень действует подобно футболисту, встречающему летящий мяч ударом ноги и сообщающему мячу скорость, значительно большую той, которой он обладал до удара.

Если газ, напротив, расширяется, то после столкновения с удаляющимся поршнем скорости молекул уменьшаются, в результате чего газ охлаждается. Так же действует футболист, для того чтобы уменьшить скорость летящего мяча или остановить его; нога футболиста движется от мяча, как бы уступая ему дорогу.

При сжатии или расширении меняется и средняя потенциальная энергия взаимодействия молекул, так как при этом меняется среднее расстояние между молекулами.

Вычисление работы. Вычислим работу в зависимости от изменения объема на примере газа в цилиндре под поршнем (рис. 39). Проще всего вначале вычислить не работу силы F, действующей на газ со стороны внешнего тела (поршня), а работу, которую совершает сам газ, действуя на поршень с силой F’. Согласно третьему закону Ньютона F’ = –F.

Модуль силы, действующей со стороны газа на поршень, равен: F’ = pS, где p – давление газа, а S – площадь поршня. Пусть газ расширяется и поршень смещается в направлении силы F’ на малое расстояние ∆h = h2 – h1. Если перемещение мало, то давление газа можно считать постоянным.

Работа газа равна:

A’ = F’∆h = pS(h2 – h1) = p(Sh2 – Sh1).     (4.2)

Эту работу можно выразить через изменение объема газа. Начальный объем V1 = Sh1, а конечный V2 = Sh2. Поэтому

A’ = p(V2 – V1) = p∆V,      (4.3)

где ∆V = V2 – V1 — изменение объема газа.

При расширении газ совершает положительную работу, так как направление силы и направление перемещения поршня совпадают. В процессе расширения газ передает энергию окружающим телам.

Если газ сжимается, то формула (4.3) для работы газа остается справедливой. Но теперь V21 и поэтому A’

Работа A, совершаемая внешними телами над газом, отличается от работы газа A’ только знаком: A = –A’, так как сила F, действующая на газ, направлена против силы F’, а перемещение остается тем же самым. Поэтому работа внешних сил, действующих на газ, равна:

A = –A’ = –p∆V     (4.4)

Знак минус указывает, что при сжатии газа, когда ∆V = V2 – V1 0: при сжатии газа направления силы и перемещения совпадают. Совершая над газом положительную работу, внешние тела передают ему энергию. При расширении газа, наоборот, работа внешних тел отрицательна (A 2 – V1 > 0. Теперь направления силы и перемещения противоположны.

Выражения (4.3) и (4.4) справедливы не только при сжатии или расширении газа в цилиндре, но и при малом изменении объема любой системы. Если процесс изобарный (p = const), то эти формулы можно применять и для больших изменений объема.

Геометрическое истолкование работы. Работе A’ газа для случая постоянного давления можно дать простое геометрическое истолкование.

Построим график зависимости давления газа от объема (рис. 41) . Здесь площадь прямоугольника abcd, ограниченная графиком p1 = const, осью V и отрезками ab и cd, равными давлению газа, численно равна работе (4.3).

В общем случае при произвольном изменении объема газа давление не остается неизменным. Например, при изотермическом процессе оно убывает обратно пропорционально объему (рис. 42). В этом случае для вычисления работы нужно разделить общее изменение объема на малые части, вычислить элементарные (малые) работы, а потом все их сложить. Работа газа по-прежнему будет численно равна площади фигуры, ограниченной графиком зависимости p от V, осью V и отрезками ab и cd, равными давлениям p1, p2 в начальном и конечном состояниях.

1. От каких физических величин зависит внутренняя энергия тела? 2. Приведите примеры превращения механической энергии во внутреннюю и обратно в темнике и быту. 3. Чему равна внутренняя энергия идеального одноатомного газа? 4. Моль какого газа – водорода или гелия – имеет большую внутреннюю энергию при одинаковой температуре газов? 5. Почему газ при сжатии нагревается? 6. Чему равна работа, совершаемая внешними силами при сжатии и расширении тел?

Формула внутренней энергии в физике

Содержание:

Определение и формула внутренней энергии

Определение

Внутренней энергией тела (системы) называют энергию, которая связана со всеми видами движения и взаимодействия частиц, составляющих тело (систему), включая энергию взаимодействия и движения сложных частиц.

Из выше сказанного следует, что к внутренней энергии не относят кинетическую энергию движения центра масс системы и потенциальную энергию системы, вызванную действием внешних сил. Это энергия, которая зависит только от термодинамического состояния системы.

Внутреннюю энергию чаще всего обозначают буквой U. При этом бесконечно малое ее изменение станет обозначаться dU. Считается, что dU является положительной величиной, если внутренняя энергия системы растет, соответственно, внутренняя энергия отрицательна, если внутренняя энергия уменьшается.

Внутренняя энергия системы тел равна сумме внутренних энергий каждого отдельного тела плюс энергия взаимодействия между телами внутри системы.

Внутренняя энергия – функция состояния системы. Это означает, что изменение внутренней энергии системы при переходе системы из одного состояния в другое не зависит от способа перехода (вида термодинамического процесса при переходе) системы и равно разности внутренних энергий конечного и начального состояний:

$$\Delta U=U_{2}-U_{1}(1)$$

Для кругового процесса полное изменение внутренней энергии системы равно нулю:

$$\oint d U=0(2)$$

Для системы, на которую не действуют внешние силы и находящуюся в состоянии макроскопического покоя, внутренняя энергия – полная энергия системы.

Внутренняя энергия может быть определена только с точностью до некоторого постоянного слагаемого (U0), которое не определимо методами термодинамики.{T} c_{V} d T+u_{0}\right)$$

где CV – теплоемкость газа в изохорном процессе; cV – удельная теплоемкость газа в изохорном процессе; $u_{0}=\frac{U_{0}}{m}$ – внутренняя энергия, приходящаяся на единицу массы газа при абсолютном нуле температур. Или:

$$d U=\frac{i}{2} \nu R d T(4)$$

i – число степеней свободы молекулы идеального газа, v – число молей газа, R=8,31 Дж/(моль•К) – универсальная газовая постоянная.

Первое начало термодинамики

Как известно первое начало термодинамики имеет несколько формулировок. Одна из формулировок, которую предложил К. Каратеодори говорит о существовании внутренней энергии как составляющей полной энергии системы.Она является функцией состояния, в простых системах зависящей от объема (V), давления (p), масс веществ (mi), которые составляют данную систему: $U=U\left(p, V, \sum m_{i}\right)$ . В формулировке, которую дал Каратеодори внутренняя энергия не является характеристической функцией своих независимых переменных.

В более привычных формулировках первого начала термодинамики, например, формулировке Гельмгольца внутренняя энергия системы вводится как физическая характеристика системы. При этом поведение системы определено законом сохранения энергии. Гельмгольц не определяет внутреннюю энергию как функцию конкретных параметров состояния системы:

$$\Delta U=Q-A(5)$$

$\Delta U$ – изменение внутренней энергии в равновесном процессе, Q – количество теплоты, которое получила система в рассматриваемом процессе, A – работа, которую система совершила.

Единицы измерения внутренней энергии

Основной единицей измерения внутренней энергии в системе СИ является: [U]=Дж

Примеры решения задач

Пример

Задание. Вычислите, на какую величину изменится внутренняя энергия гелия имеющего массу 0,1 кг, если его температура увеличилась на 20С.

Решение. При решении задачи считаем гелий одноатомным идеальным газом, тогда для расчетов можно применить формулу:

$$d U=\frac{i}{2} \nu R d T(1.{3}$ (Дж)

Слишком сложно?

Формула внутренней энергии не по зубам? Тебе ответит эксперт через 10 минут!

Пример

Задание. Идеальный газ расширили в соответствии с законом, который изображен графиком на рис.1. от начального объема V0. При расширении объем сал равен $V=\tau V_{0}$ . Каково приращение внутренней энергии газа в заданном процессе? Коэффициент адиабаты равен $\gamma$.

Решение. Исходя из рисунка, уравнение процесса можно представить аналитически как:

$$p=\alpha V(2.1)$$

Показатель адиабаты связан с числом степеней свободы газа выражением:

$$\gamma=\frac{i+2}{i}(2.2)$$

Выразим число степеней свободы из (2.2):

$$i=\frac{2}{\gamma-1}$$

Приращение внутренней энергии для постоянной массы газа (см. Пример 1) найдем в соответствии с формулой:

$$\Delta U=\frac{i}{2} \nu R \Delta T(2.4)$$

Запишем уравнения состояний идеального газа для точек (1) и (2) рис.{2}-1\right)$

Читать дальше: Формула времени.

Первый закон термодинамики

Цели обучения

К концу этого раздела вы сможете:

  • Определите первый закон термодинамики.
  • Опишите, как сохранение энергии соотносится с первым законом термодинамики.
  • Выявить примеры первого закона термодинамики, работающие в повседневных ситуациях, включая биологический метаболизм.
  • Рассчитывает изменения внутренней энергии системы после учета теплопередачи и проделанной работы.

Рис. 1. Этот кипящий чайник представляет энергию в движении. Вода в чайнике превращается в водяной пар, потому что тепло передается от плиты к чайнику. По мере того, как вся система нагревается, работа выполняется – от испарения воды до свиста чайника. (кредит: Джина Гамильтон)

Если нас интересует, как теплопередача преобразуется в работу, тогда важен принцип сохранения энергии. Первый закон термодинамики применяет принцип сохранения энергии к системам, в которых передача тепла и выполнение работы являются методами передачи энергии в систему и из нее.Первый закон термодинамики гласит, что изменение внутренней энергии системы равно чистой теплопередаче в системе за вычетом чистой работы, выполненной системой. В форме уравнения первый закон термодинамики: Δ U = Q W .

Здесь Δ U – это изменение внутренней энергии U системы. Q – это чистое тепло , переданное в систему. , то есть Q – это сумма всей теплопередачи в систему и из нее. W – это чистая работа , выполненная системой , то есть W – это сумма всей работы, выполненной в системе или ею. Мы используем следующие условные обозначения: если значение Q положительно, то в системе имеется чистый теплоперенос; если значение W положительное, значит, система выполняет чистую работу. Таким образом, положительный Q добавляет энергию в систему, а положительный W забирает энергию из системы. Таким образом Δ U = Q Вт .Также обратите внимание, что если в систему передается больше тепла, чем проделанной работы, разница сохраняется как внутренняя энергия. Тепловые двигатели – хороший тому пример – в них происходит передача тепла, чтобы они могли выполнять свою работу. (См. Рисунок 2.) Теперь мы рассмотрим Q , W и Δ U далее.

Рис. 2. Первый закон термодинамики – это принцип сохранения энергии, установленный для системы, в которой тепло и работа являются методами передачи энергии для системы, находящейся в тепловом равновесии. Q представляет собой чистую теплопередачу – это сумма всех теплопередач в систему и из нее. Q положителен для чистой передачи тепла в систему. W – это общий объем работы, проделанной с системой. W положителен, когда система выполняет больше работы, чем над ней. Изменение внутренней энергии системы Δ U связано с теплом и работой по первому закону термодинамики Δ U = Q Вт .

Установление связей: закон термодинамики и закон сохранения энергии

Первый закон термодинамики – это фактически закон сохранения энергии, сформулированный в форме, наиболее полезной в термодинамике. Первый закон устанавливает связь между теплопередачей, проделанной работой и изменением внутренней энергии системы.

Нагрев

Q и рабочий Вт

Теплопередача ( Q ) и выполнение работы ( W ) – два повседневных средства подачи энергии в систему или вывода энергии из системы.Процессы совершенно разные. Теплообмен, менее организованный процесс, обусловлен разницей температур. Работа – это вполне организованный процесс, в котором действует макроскопическая сила, действующая на расстоянии. Тем не менее, тепло и работа могут дать одинаковые результаты, например, оба могут вызвать повышение температуры. Передача тепла в систему, например, когда Солнце нагревает воздух в велосипедной шине, может повысить ее температуру, и поэтому может работать над системой, например, когда велосипедист нагнетает воздух в шину.Как только произошло повышение температуры, невозможно сказать, было ли оно вызвано теплопередачей или работой. Эта неопределенность – важный момент. Теплообмен и работа – это энергия в пути, и ни одна из них не хранится как таковая в системе. Однако оба могут изменить внутреннюю энергию U системы. Внутренняя энергия – это форма энергии, полностью отличная от тепла или работы.

Внутренняя энергия

U

Мы можем думать о внутренней энергии системы двумя разными, но последовательными способами.Первый – это атомно-молекулярная точка зрения, которая исследует систему в атомном и молекулярном масштабе. Внутренняя энергия U системы – это сумма кинетической и потенциальной энергий ее атомов и молекул. Напомним, что кинетическая плюс потенциальная энергия называется механической энергией. Таким образом, внутренняя энергия – это сумма атомной и молекулярной механической энергии. Поскольку невозможно отследить все отдельные атомы и молекулы, мы должны иметь дело со средними значениями и распределениями.Второй способ взглянуть на внутреннюю энергию системы – с точки зрения ее макроскопических характеристик, которые очень похожи на средние атомные и молекулярные значения.

Макроскопически мы определяем изменение внутренней энергии Δ U как значение, определяемое первым законом термодинамики: Δ U = Q Вт .

Многие подробные эксперименты подтвердили, что Δ U = Q W , где Δ U – изменение полной кинетической и потенциальной энергии всех атомов и молекул в системе.Также экспериментально было определено, что внутренняя энергия U системы зависит только от состояния системы, а не от того, как она достигла этого состояния . Более конкретно, U оказывается функцией нескольких макроскопических величин (например, давления, объема и температуры), независимо от прошлой истории, например от того, была ли проведена теплопередача или была проделана работа. Эта независимость означает, что, зная состояние системы, мы можем рассчитать изменения ее внутренней энергии U на основе нескольких макроскопических переменных.

Установление соединений: макроскопическое и микроскопическое

В термодинамике мы часто используем макроскопическую картину при расчетах поведения системы, в то время как атомная и молекулярная картина дает основные объяснения в терминах средних значений и распределений. Мы еще раз увидим это в следующих разделах этой главы. Например, в теме энтропии расчеты будут производиться с использованием атомно-молекулярного представления.

Чтобы лучше понять, как думать о внутренней энергии системы, давайте рассмотрим систему, переходящую из состояния 1 в состояние 2.Система имеет внутреннюю энергию U 1 в Состоянии 1 и внутреннюю энергию U 2 в Состоянии 2, независимо от того, как она попала в любое из состояний. Таким образом, изменение внутренней энергии Δ U = U 2 U 1 не зависит от того, что вызвало изменение. Другими словами, Δ U не зависит от пути . Под путем мы подразумеваем способ добраться от начальной точки до конечной точки. Почему важна эта независимость? Обратите внимание, что Δ U = Q W Q , и W зависят от пути , а Δ U – нет. Эта независимость от пути означает, что внутреннюю энергию U легче учитывать, чем теплопередачу или проделанную работу.

Пример 1. Расчет изменения внутренней энергии: одно и то же изменение в

U производится двумя разными процессами
  1. Предположим, что теплопередача в систему составляет 40,00 Дж, в то время как система выполняет работу 10,00 Дж. Позже появляется теплоотдача 25.00 Дж вне системы, в то время как в системе выполняется 4,00 Дж работы. Каково чистое изменение внутренней энергии системы?
  2. Каково изменение внутренней энергии системы, когда в общей сложности 150,00 Дж теплопередачи происходит из (от) системы и 159,00 Дж работы выполняется в системе? (См. Рисунок 3).

Рисунок 3. Два разных процесса производят одно и то же изменение в системе. (a) Всего в системе происходит передача тепла 15,00 Дж, в то время как работа требует всего 6.00 Дж. Изменение внутренней энергии составляет ΔU = Q-W = 9,00 Дж. (B) При передаче тепла из системы удаляется 150,00 Дж, в то время как работа затрачивает в нее 159,00 Дж, что приводит к увеличению внутренней энергии на 9,00 Дж. Если система начинает работу в одном и том же состоянии в пунктах (а) и (б), она окажется в одном и том же конечном состоянии в любом случае – ее конечное состояние связано с внутренней энергией, а не с тем, как эта энергия была получена.

Стратегия

В части 1 мы должны сначала найти чистую теплопередачу и чистую работу, выполненную на основе данной информации.Тогда первый закон термодинамики (Δ U = Q W ) может быть использован для определения изменения внутренней энергии. В части (b) приведены чистая теплопередача и проделанная работа, поэтому уравнение можно использовать напрямую.

Решение для Части 1

Чистая теплопередача – это теплопередача в систему за вычетом теплопередачи из системы, или

Q = 40,00 Дж – 25,00 Дж = 15,00 Дж

Аналогично, общая работа – это работа, выполненная системой за вычетом работы, выполненной в системе, или

Вт = 10.00 Дж – 4,00 Дж = 6,00 Дж.

Таким образом, изменение внутренней энергии определяется первым законом термодинамики:

Δ U = Q W = 15,00 Дж – 6,00 Дж = 9,00 Дж

Мы также можем найти изменение внутренней энергии для каждого из двух шагов. Сначала рассмотрим 40,00 Дж теплопередачи на входе и 10,00 Дж на выходе, или Δ U 1 = Q 1 W 1 = 40,00 Дж – 10,00 Дж = 30,00 Дж.

Теперь рассмотрим 25,00 Дж теплоотдачи и 4,00 Дж работы на входе, или

Δ U 2 = Q 2 W 2 = –25,00 Дж – (- 4,00 Дж) = –21,00 Дж

Общее изменение – это сумма этих двух шагов, или Δ U = Δ U 1 + Δ U 2 = 30,00 Дж + (−21,00 Дж) = 9,00 Дж.

Обсуждение части 1

Неважно, смотрите ли вы на процесс в целом или разбиваете его на этапы, изменение внутренней энергии одинаково.

Решение для Части 2

Здесь чистая теплопередача и общая работа даны непосредственно как Q = –150,00 Дж и Вт = –159,00 Дж, так что

Δ U = Q W = –150,00 Дж – (- 159,00 Дж) = 9,00 Дж.

Обсуждение части 2

Совершенно другой процесс в части 2 дает такое же изменение внутренней энергии на 9,00 Дж, что и в части 1. Обратите внимание, что изменение в системе в обеих частях связано с Δ U , а не с отдельным Q s или Вт с задействовано.Система оказывается в состоянии , одинаковое в обеих частях. Части 1 и 2 представляют два разных пути, которыми должна следовать система между одними и теми же начальными и конечными точками, и изменение внутренней энергии для каждой из них одинаково – оно не зависит от пути.

Метаболизм человека и первый закон термодинамики

Метаболизм человека – это преобразование пищи в теплообмен, работу и накопленный жир. Метаболизм – интересный пример действия первого закона термодинамики.Теперь мы еще раз посмотрим на эти темы с помощью первого закона термодинамики. Рассматривая тело как интересующую нас систему, мы можем использовать первый закон для изучения теплопередачи, выполнения работы и внутренней энергии в различных видах деятельности, от сна до тяжелых упражнений. Каковы некоторые из основных характеристик теплопередачи, выполнения работы и энергии в организме? Во-первых, температура тела обычно поддерживается постоянной за счет передачи тепла в окружающую среду. Это означает, что Q отрицательный. Другой факт: тело обычно работает с внешним миром.Это означает, что W положительный. В таких ситуациях тело теряет внутреннюю энергию, поскольку Δ U = Q Вт отрицательно.

Теперь рассмотрим эффекты еды. Прием пищи увеличивает внутреннюю энергию тела за счет добавления химической потенциальной энергии (это неромантичный взгляд на хороший стейк). Тело метаболизирует всю пищу, которую мы потребляем. По сути, метаболизм – это процесс окисления, при котором высвобождается химическая потенциальная энергия пищи.Это означает, что питание осуществляется в форме работы. Энергия пищи указывается в специальной единице, известной как калория. Эта энергия измеряется сжиганием пищи в калориметре, как и определяются единицы.

В химии и биохимии одна калория (обозначается строчной c) определяется как энергия (или передача тепла), необходимая для повышения температуры одного грамма чистой воды на один градус Цельсия. Диетологи и любители веса склонны использовать диетических калорий, которые часто называют калориями (пишется с заглавной C).Одна еда Калория – это энергия, необходимая для повышения температуры одного килограмма воды на один градус Цельсия. Это означает, что одна диетическая калория для химика равна одной килокалории, и нужно быть осторожным, чтобы не путать их.

Опять же, рассмотрим внутреннюю энергию, потерянную телом. Эта внутренняя энергия может идти по трем направлениям – на теплопередачу, выполнение работы и накопленный жир (крошечная часть также идет на восстановление и рост клеток). Передача тепла и выполнение работы забирают внутреннюю энергию из организма, а пища возвращает ее.Если вы едите нужное количество еды, ваша средняя внутренняя энергия остается постоянной. Все, что вы теряете на теплопередачу и выполнение работы, заменяется едой, так что в конечном итоге Δ U = 0. Если вы постоянно переедаете, то Δ U всегда положительный, и ваше тело сохраняет эту дополнительную внутреннюю энергию в виде жира. Обратное верно, если вы едите слишком мало. Если Δ U будет отрицательным в течение нескольких дней, то организм усваивает собственный жир, чтобы поддерживать температуру тела и выполнять работу, которая забирает у тела энергию.Именно так соблюдение диеты способствует снижению веса.

Жизнь не всегда так проста, как знает любой человек, сидящий на диете. Тело накапливает жир или метаболизирует его только в том случае, если потребление энергии меняется в течение нескольких дней. После того, как вы сели на основную диету, следующая будет менее успешной, потому что ваше тело изменит способ реагирования на низкое потребление энергии. Ваша основная скорость метаболизма (BMR) – это скорость, с которой пища преобразуется в теплообмен и работу, выполняемую, когда организм находится в полном покое. Организм регулирует базальную скорость метаболизма, чтобы частично компенсировать переедание или недоедание.Организм будет снижать скорость метаболизма, а не устранять собственный жир, чтобы заменить потерянную еду. Вы легче простужаетесь и чувствуете себя менее энергичным из-за более низкой скорости метаболизма, и вы не будете терять вес так быстро, как раньше. Упражнения помогают похудеть, потому что они обеспечивают теплоотдачу от вашего тела и работы, а также повышают уровень метаболизма, даже когда вы находитесь в состоянии покоя. Снижению веса также способствует довольно низкая эффективность тела в преобразовании внутренней энергии в работу, так что потеря внутренней энергии в результате выполнения работы намного больше, чем проделанная работа.Однако следует отметить, что живые системы не находятся в тепловом равновесии.

Тело дает нам отличный показатель того, что многие термодинамические процессы необратимы . Необратимый процесс может идти в одном направлении, но не в обратном, при заданном наборе условий. Например, хотя телесный жир может быть преобразован для выполнения работы и передачи тепла, работа, выполняемая телом, и передача тепла в него не могут быть преобразованы в телесный жир. В противном случае мы могли бы пропустить обед, загорая или спустившись по лестнице.Другой пример необратимого термодинамического процесса – фотосинтез. Этот процесс представляет собой поглощение растениями одной формы энергии – света – и ее преобразование в химическую потенциальную энергию. Оба применения первого закона термодинамики показаны на рисунке 4. Одно большое преимущество законов сохранения, таких как первый закон термодинамики, состоит в том, что они точно описывают начальную и конечную точки сложных процессов, таких как метаболизм и фотосинтез, без учета осложнения между ними.В таблице 1 представлена ​​сводка терминов, относящихся к первому закону термодинамики.

Рис. 4. (а) Первый закон термодинамики применительно к метаболизму. Тепло, передаваемое из тела (Q), и работа, выполняемая телом (W), удаляют внутреннюю энергию, в то время как прием пищи заменяет ее. (Прием пищи можно рассматривать как работу, выполняемую телом.) (Б) Растения преобразуют часть лучистой теплопередачи в солнечном свете в запасенную химическую энергию – процесс, называемый фотосинтезом.

Таблица 1.Краткое изложение терминов первого закона термодинамики, ΔU = Q – W
Срок Определение
U Внутренняя энергия – сумма кинетической и потенциальной энергий атомов и молекул системы. Можно разделить на множество подкатегорий, таких как тепловая и химическая энергия. Зависит только от состояния системы (например, P , V и T ), а не от того, как энергия поступает в систему.Изменение внутренней энергии не зависит от пути.
Q Тепло – энергия, передаваемая из-за разницы температур. Характеризуется случайным движением молекул. Сильно зависит от пути. Q Вход в систему положительный.
Вт Работа – энергия, передаваемая силой, перемещающейся на расстояние. Организованный, упорядоченный процесс. Зависит от пути. W , выполненный системой (либо против внешней силы, либо для увеличения объема системы), является положительным.

Сводка раздела

  • Первый закон термодинамики задается как Δ U = Q – W , где Δ U – изменение внутренней энергии системы, Q – чистая теплопередача (сумма вся теплопередача в систему и из нее), а W – это чистая проделанная работа (сумма всей работы, проделанной в системе или ею).
  • И Q , и W – энергия в пути; только Δ U представляет собой независимую величину, которую можно хранить.
  • Внутренняя энергия U системы зависит только от состояния системы, а не от того, как она достигла этого состояния.
  • Метаболизм живых организмов и фотосинтез растений – это особые виды передачи тепла, выполнения работы и внутренней энергии систем.

Концептуальные вопросы

  1. Опишите фотографию чайника в начале этого раздела с точки зрения теплопередачи, проделанной работы и внутренней энергии. Как передается тепло? Какая работа и что делается? Как чайник поддерживает свою внутреннюю энергию?
  2. Первый закон термодинамики и закон сохранения энергии, как обсуждалось в «Сохранении энергии», явно связаны.Чем они различаются по рассматриваемым видам энергии?
  3. Теплопередача Q и выполненная работа Вт – это всегда энергия в пути, тогда как внутренняя энергия U – это энергия, запасенная в системе. Приведите пример каждого типа энергии и конкретно укажите, как он передается или находится в системе.
  4. Чем отличаются теплопередача и внутренняя энергия? В частности, что можно сохранить как таковое в системе, а что нет?
  5. Если вы сбежите по лестнице и остановитесь, что произойдет с вашей кинетической энергией и вашей начальной гравитационной потенциальной энергией?
  6. Объясните, как пищевая энергия (калории) может рассматриваться как молекулярная потенциальная энергия (в соответствии с атомарным и молекулярным определением внутренней энергии).
  7. Определите тип энергии, передаваемой вашему телу в каждом из следующих случаев: внутренняя энергия, теплопередача или выполнение работы: (а) купание в солнечном свете; (б) употребление пищи; (c) подъем на лифте на более высокий этаж.

Задачи и упражнения

  1. Как изменится внутренняя энергия автомобиля, если в его бак залить 12 галлонов бензина? Энергетическая ценность бензина составляет 1,3 × 10 8 Дж / галлон. Все остальные факторы, например температура в автомобиле, постоянны.
  2. Сколько тепла происходит от системы, если ее внутренняя энергия уменьшилась на 150 Дж, пока она выполняла 30,0 Дж работы?
  3. Система выполняет 1,80 × 10 8 Дж работы, в то время как 7,50 × 10 8 Дж теплопередачи происходит в окружающую среду. Каково изменение внутренней энергии системы при отсутствии других изменений (например, температуры или добавления топлива)?
  4. Каково изменение внутренней энергии системы, которая выполняет 4,50 × 10 5 Дж, пока 3.00 × 10 6 Дж происходит теплопередача в системе, а 8.00 × 10 6 Дж теплопередачи происходит в окружающую среду?
  5. Предположим, что женщина выполняет 500 Дж работы, и 9500 Дж в процессе передачи тепла в окружающую среду. а) Как уменьшается ее внутренняя энергия, если не меняется температура или потребление пищи? (То есть другой передачи энергии нет.) Б) Какова ее эффективность?
  6. (а) Сколько пищевой энергии человек усвоит в процессе усвоения 35.0 кДж работы при КПД 5,00%? б) Сколько тепла передается в окружающую среду, чтобы поддерживать постоянную температуру?
  7. (а) Какова средняя скорость метаболизма в ваттах человека, который усваивает 10 500 кДж пищевой энергии за один день? (б) Какое максимальное количество работы в джоулях он может выполнить без расщепления жира, предполагая максимальную эффективность 20,0%? (c) Сравните его производительность с дневной мощностью двигателя 187 Вт (0,250 лошадиных сил).
  8. (a) На сколько хватит энергии в стакане йогурта мощностью 1470 кДж (350 ккал) у женщины, выполняющей работу с мощностью 150 Вт с эффективностью 20?0% (например, при неспешном подъеме по лестнице)? (б) Означает ли время, указанное в части (а), легко потребить больше пищевой энергии, чем вы можете разумно ожидать, работая с упражнениями?
  9. (a) Женщина, поднимающаяся на памятник Вашингтону, усваивает 6,00 × 10 2 кДж пищевой энергии. Если ее КПД составляет 18,0%, сколько тепла передается в окружающую среду, чтобы поддерживать ее температуру постоянной? (б) Обсудите величину теплопередачи, указанную в (а). Это согласуется с тем, что вы быстро разминаетесь во время тренировки?

Глоссарий

Первый закон термодинамики: утверждает, что изменение внутренней энергии системы равно чистой теплопередаче в систему за вычетом чистой работы, выполненной системой

внутренняя энергия: сумма кинетической и потенциальной энергий атомов и молекул системы

метаболизм человека: преобразование пищи в теплообмен, работу и накопленный жир

Избранные решения проблем и упражнения

1.1,6 × 10 9 Дж

3. −9.30 × 10 8 Дж

5. (а) -1,0 × 10 4 Дж, или -2,39 ккал; (б) 5,00%

7. (а) 122 Вт; (б) 2,10 × 10 6 Дж; (c) Работа двигателя составляет 1,61 × 10 7 Дж; таким образом, двигатель производит в 7,67 раз больше работы, чем человек

9. (а) 492 кДж; (б) Такое количество тепла соответствует тому факту, что вы быстро согреваетесь во время тренировки. Поскольку организм неэффективен, выделяемое избыточное тепло должно рассеиваться через потоотделение, дыхание и т. Д.

Уравнения термодинамики

Химическая система будет иметь тенденцию переходить из одного состояния в другое любыми доступными средствами, так что универсальная энтропия увеличивается. Максимальное универсальное изменение энтропии будет совпадать с конечным состоянием, которое имеет тенденцию иметь наименьшую возможную полную свободную энергию из всех компонентов. Если это состояние достигается, то при таком точном составе все реагенты имеют точную полную свободную энергию, как и все продукты. Это также означает, что ∆ G = 0 в этой точке.Это точка, в которой было достигнуто равновесие. Нет больше никакой тенденции для реакции идти в чистом прямом направлении или чистом обратном направлении. Любой способ фактически увеличил бы свободную энергию системы. Состояние равновесия – это в основном то, чего «пытаются» достичь все реакции. «Стимул» к достижению этого положения / состояния – это изменение свободной энергии реакции ∆ G . Если это количество положительное, тогда реакция будет происходить в обратном направлении от того, как написано на странице – несамопроизвольное прямое направление означает спонтанное обратное направление.Если величина отрицательная (-∆ G ), тогда реакция будет ускорена.

И последнее … по мере развития реакции количества реагентов и продуктов постоянно меняются. Количество реагентов уменьшается, а количество продуктов увеличивается. Осознайте, что свободная энергия связана с количеством вещества, а также с тем, что это за вещество. Когда соединение уменьшается, его свободная энергия также уменьшается, потому что ее становится все меньше и меньше.Если все это отреагирует, то свободная энергия этого вещества теперь равна нулю, потому что его больше нет. Конечно, по другую сторону стрелки – товары. Когда началась реакция, у них была нулевая свободная энергия (потому что там ничего не было), но затем начинают появляться продукты, и свободная энергия растет. Реакция фактически остановится, когда сумма всех свободных энергий реагентов сравняется с полной свободной энергией продуктов. Если свободные энергии совпадают, то ∆ G должно быть равно нулю, и система достигла равновесия.

Так что это значит? Что ж, независимо от того, какой у вас ∆ G для реакции – будь то положительное или отрицательное, абсолютное значение ∆ G будет уменьшаться и продолжать уменьшаться по мере того, как реакция идет вперед (или назад). Как только реакция достигает той точки, где ∆ G = 0, равновесие теперь «в игре». Когда вы достигнете равновесия, большое значение приобретет множество замечательных отношений и уравнений. Все это “чудо” мы сохраняем для вас в Ch402.

Прочтите предыдущие абзацы снова и снова и ПОПРОБУЙТЕ, чтобы понять, о чем они говорят. Понимание этого – КЛЮЧ к хорошему пониманию того, что движет всем вокруг нас. Термодинамика управляет движущей силой всех вещей.

Термодинамика | Физика для идиотов

Есть 4 закона термодинамики, и это одни из самых важных законов во всей физике. Законы следующие

  • Нулевой закон термодинамики – Если две термодинамические системы каждая находится в тепловом равновесии с третьей, то они находятся в тепловом равновесии друг с другом.
  • Первый закон термодинамики – Энергия не может быть ни создана, ни разрушена. Он может менять только формы. В любом процессе общая энергия Вселенной остается неизменной. Для термодинамического цикла чистое тепло, подаваемое в систему, равно чистой работе, выполненной системой.
  • Второй закон термодинамики – Энтропия изолированной системы, не находящейся в равновесии, будет со временем увеличиваться, приближаясь к максимальному значению в состоянии равновесия.
  • Третий закон термодинамики – Когда температура приближается к абсолютному нулю, энтропия системы приближается к постоянному минимуму.

Прежде чем я рассмотрю эти законы более подробно, будет проще, если я сначала расскажу об энтропии.

Энтропия – очень важная вещь в области термодинамики. Это основная идея второго и третьего законов, которая проявляется повсюду. По сути, энтропия – это мера беспорядка и случайности в системе. Вот 2 примера

  • Допустим, у вас есть контейнер с молекулами газа. Если все молекулы находятся в одном углу, это будет состояние с низкой энтропией (высокоорганизованное).По мере того, как частица перемещается и заполняет остальную часть контейнера, энтропия (беспорядок) увеличивается.
  • Если у вас есть мяч, летящий по воздуху, он начнет движение с организованной энергией, то есть кинетической энергией движения. Однако, когда он движется по воздуху, часть кинетической энергии распределяется между частицами воздуха, поэтому общая энтропия системы увеличивается (однако общая энергия сохраняется благодаря первому закону)

Чтобы получить более подробную картину энтропии, нам нужно взглянуть на концепцию фазового пространства.Некоторые из концепций этого могут быть немного запутанными, но потерпите меня, если вы разберетесь с мыслями, все не так уж и плохо.

Фазовое пространство похоже на график, но точка на этом графике представляет все состояние системы. Приведем пример. Представьте, что у меня есть ящик с 4 частицами газа внутри. Каждая точка в фазовом пространстве этой системы сообщает вам, где находятся все 4 шара в коробке.

В нашем примере нас интересуют только положения 4 частиц, поэтому каждая точка в фазовом пространстве должна содержать координаты x, y и z для каждой частицы, чтобы наше фазовое пространство было 3N-мерным, где N – число частиц в системе.Итак, в нашем случае фазовое пространство 12-мерное, чтобы каждая точка могла описывать расположение 4 тел.

На всех диаграммах я буду изображать фазовое пространство как 2D, чтобы было легче передать то, что оно на самом деле представляет. Для наших целей нам не нужно учитывать размеры.

Если мы представим, что каждая из частиц имеет разный цвет, мы сможем легче отслеживать их положение. Если мы представим случай, когда все частицы расположены в одном углу контейнера, то у нас будет ситуация

С точки зрения системы существует множество других комбинаций из 4 частиц, которые будут иметь такую ​​же организацию, как и состояние выше

и так далее.Каждая из этих установок будет соответствовать разному положению в фазовом пространстве, поскольку все они представляют собой различные схемы системы из 4-х частиц. Если мы добавим их в фазовое пространство вместе с оригиналом, мы получим что-то вроде

.

Эти 5 макетов из 4 частиц вместе с 11 другими комбинациями составляют набор состояний, которые (кроме цветов) неразличимы. Итак, в фазовом пространстве мы могли бы поместить рамку вокруг 16 состояний, которая определяет все состояния внутри него как макроскопически неразличимые.

Общее фазовое пространство системы будет иметь множество областей разной формы и размера и может выглядеть следующим образом:

Но как все это абстрактное представление связано с энтропией? Энтропия, указанная в уравнениях как символ, определяется как

.

Где – постоянная Больцмана (), а – объем прямоугольника в фазовом пространстве. Все точки в области фазового пространства имеют одинаковую энтропию, и значение энтропии связано с логарифмом объема (изначально Больцман никогда не вставлял константу в формулу, поскольку его не интересовали единицы измерения.Буква k, по-видимому, впервые была введена Планком).

Энтропию также можно определить как изменение при передаче энергии при постоянной температуре

Где – изменение энтропии, – энергия или тепло, а T – постоянная температура.

Нулевой закон назван так, потому что он появился после остальных 3. Законы 1, 2 и 3 существовали некоторое время, прежде чем важность этого закона была полностью осознана. Оказалось, что этот закон был настолько важен и фундаментален, что он должен был пойти раньше остальных трех, и вместо того, чтобы переименовать уже известные три закона, они назвали новый закон Нулевым и поместили его в начале списка.

Но что это на самом деле означает? В законе указано

«Если две термодинамические системы каждая находится в тепловом равновесии с третьей, то они находятся в тепловом равновесии друг с другом».

В принципе, если A = B и C = B, то A = C. Это может показаться настолько очевидным, что об этом не нужно говорить, но без этого закона мы не смогли бы определить температуру и не смогли бы построить термометры.

Первый закон термодинамики в основном утверждает, что энергия сохраняется; его нельзя ни создать, ни уничтожить, просто поменять с одного на другое,

«Общее количество энергии в изолированной системе сохраняется.”

Энергия в системе может быть преобразована в тепло, работу или другие вещи, но у вас всегда будет та же сумма, с которой вы начали.

В качестве аналогии подумайте об энергии как о неразрушимых блоках. Если у вас есть 30 блоков, то что бы вы ни делали с блоками или с ними, у вас всегда будет 30 из них в конце. Вы не можете уничтожить их, только перемещайте их или разделяйте, но их всегда будет 30. Иногда вы можете потерять один или несколько, но их все равно нужно учитывать, потому что энергия сохраняется.

Фундаментальное термодинамическое соотношение

Из второго закона мы можем написать, что изменение внутренней энергии, системы равно теплу, подводимому к системе, минус любая работа, выполняемая системой,,

(1)

Из определения энтропии, приведенного выше, мы можем заменить, и мы также можем сделать замену, дающую нам

(2)

Теперь, если у нас есть система различных частиц, мы можем получить протекающие химические реакции, поэтому нам нужно добавить еще один член, чтобы учесть это.

(3)

Это, пожалуй, самый известный (по крайней мере, среди ученых) и важный закон всей науки.Говорится;

«Энтропия Вселенной стремится к максимуму».

Другими словами, энтропия либо остается прежней, либо увеличивается, энтропия вселенной никогда не может снизиться.

Проблема в том, что этот грех не всегда так. Если вы возьмете наш пример с 4 атомами в коробке, то все они, находящиеся в одном углу, представляют собой высокоупорядоченную систему и, следовательно, будут иметь низкую энтропию, а затем со временем они будут двигаться, становясь более неупорядоченными и увеличивая энтропию.Но ничто не мешает им беспорядочно возвращаться в угол. Это невероятно маловероятно, но на самом деле возможно.

Если вы посмотрите на проблему с точки зрения фазового пространства, вы увидите, что со временем более вероятно, что вы переместитесь в более крупную коробку, что означает более высокую энтропию, но на самом деле нет никаких препятствий, мешающих вам вернуться в меньшую коробку.

Третий закон обеспечивает абсолютную точку отсчета для измерения энтропии, говоря, что

«Когда температура системы приближается к абсолютному нулю (−273.15 ° C, 0 K), то значение энтропии приближается к минимуму ».

Значение энтропии обычно равно 0 при 0K, однако бывают случаи, когда в системе все еще остается небольшое количество остаточной энтропии.

Когда вы нагреваете что-либо, в зависимости от того, из чего оно сделано, для нагрева требуется разное время. Если предположить, что мощность, количество энергии, передаваемой за единицу времени, остается постоянной, это должно означать, что некоторым материалам требуется больше энергии для повышения их температуры на 1 К (1 К на самом деле то же самое, что 1 ° C, они просто начинаются в другом месте. .Для получения дополнительной информации щелкните здесь), чем другие. Если задуматься, в этом есть смысл. Деревянная ложка нагревается намного дольше, чем металлическая. Мы говорим, что металл является хорошим проводником тепла, а дерево – плохим проводником тепла. Энергия, необходимая для подъема 1 кг вещества на 1 К, называется удельной теплоемкостью. Формула, которую мы используем, чтобы найти, сколько энергии требуется для подъема 1 кг вещества на 1К, составляет:

где = энергия, = масса, = удельная теплоемкость и = изменение температуры.

1а. Лора готовит завтрак перед работой в воскресенье утром (пожалуйста, пришлите сюда свои сообщения сочувствия, что мне пришлось работать в воскресенье). Она больше не хочет мыть посуду, которая абсолютно необходима, поэтому решает помешать спагетти, которые готовит, вилкой, вместо того, чтобы мыть деревянной ложкой. Она оставляет вилку на сковороде, намазывает тост маргарином и натирает сыр. Плита выдает вилке 1000 Дж энергии за то время, пока она ее не трогает.Каким будет повышение температуры вилки, если предположить, что половина выделяемой энергии будет потеряна для окружающей среды, а начальная температура вилки составляла 20 ° C, а масса вилки составляет 50 г и изготовлена ​​из материала с особыми свойствами. тепловая мощность 460 Джкг-1К-1

Хотя я почти уверен, что где-то читал, что попытка проработать энергетические изменения в вилках первым делом утром была симптомом безумия, я время от времени делаю это. Для этого вопроса нам понадобится уравнение Q = mcΔT. Это уравнение, которое вам, вероятно, понадобится очень часто, поэтому стоит попытаться запомнить его.Это также возникает в химии. Прежде всего, нам нужно изменить уравнение, чтобы сделать ΔT объектом. После перестановки этого вопроса вы должны получить ΔT = Q / (mc). Подставляя данные нам значения в вопрос, вы получаете:

ΔT = 1000 / (50 x 10-3 x 460)
ΔT = 43K
Итак, поскольку начальная температура вилки была 20 ° C, конечная температура вилки будет 63 ° C.

  • Внутренняя энергия:
  • Бесплатная энергия Гельмгольца:
  • Энтальпия:
  • Свободная энергия Гиббса:

Отношения Максвелла

(4)

(5)

(6)

(7)

1-й закон термодинамики – Chemistry LibreTexts

Чтобы понять и выполнить любой вид термодинамических расчетов, мы должны сначала понять фундаментальные законы и концепции термодинамики.Например, работа и тепло – понятия взаимосвязанные. Тепло – это передача тепловой энергии между двумя телами, находящимися при разных температурах, и не равная тепловой энергии. Работа – это сила, используемая для передачи энергии между системой и ее окружением и необходимая для создания тепла и передачи тепловой энергии. И работа, и тепло вместе позволяют системам обмениваться энергией. Взаимосвязь между двумя концепциями можно проанализировать с помощью темы термодинамики, которая представляет собой научное исследование взаимодействия тепла и других типов энергии.

Введение

Чтобы понять взаимосвязь между работой и теплом, нам нужно понять третий, связывающий фактор: изменение внутренней энергии. Энергия не может быть создана или уничтожена, но ее можно преобразовать или передать. Внутренняя энергия относится ко всей энергии в данной системе, включая кинетическую энергию молекул и энергию, хранящуюся во всех химических связях между молекулами. Благодаря взаимодействию тепла, работы и внутренней энергии происходит передача и преобразование энергии каждый раз, когда в системе вносятся изменения.Однако во время этих передач чистая энергия не создается и не теряется.

Закон термодинамики

Первый закон термодинамики гласит, что энергия может быть преобразована из одной формы в другую при взаимодействии тепла, работы и внутренней энергии, но она не может быть создана или разрушена ни при каких обстоятельствах. Математически это представлено как

\ [\ Delta U = q + w \ label {1} ​​\]

с

  • \ (ΔU \) – полное изменение внутренней энергии системы,
  • \ (q \) – теплообмен между системой и ее окружением, а
  • \ (w \) – это работа, выполняемая системой или в ней.

Работа также равна отрицательному внешнему давлению в системе, умноженному на изменение объема:

\ [w = -p \ Delta V \ label {2} \]

где \ (P \) – внешнее давление на систему, а \ (ΔV \) – изменение объема. Это конкретно называется работой «давление-объем».

Внутренняя энергия системы уменьшится, если система будет выделять тепло или работать. Следовательно, внутренняя энергия системы увеличивается, когда увеличивается количество тепла (это может быть сделано путем добавления тепла в систему).Внутренняя энергия также увеличилась бы, если бы над системой выполнялась работа. Любая работа или тепло, входящие в систему или выходящие из нее, изменяют внутреннюю энергию. Однако, поскольку энергия никогда не создается и не разрушается (таким образом, первый закон термодинамики), изменение внутренней энергии всегда равно нулю. Если энергия теряется системой, она поглощается окружающей средой. Если энергия поглощается системой, то эта энергия была выпущена окружением:

\ [\ Delta U_ {system} = – \ Delta U_ {окружение} \]

, где ΔU система – полная внутренняя энергия в системе, а ΔU окружающей среды – полная энергия окружающей среды.

Таблица 1
Процесс Знак тепла (q) Знак работы (w)
Работа, выполненная системой НЕТ
Работа над системой НЕТ +
Тепло, выделяемое системой – экзотермическое (поглощаемое окружающей средой) НЕТ

Рисунок выше является наглядным примером Первого закона термодинамики.Синие кубы представляют систему, а желтые круги – окружение системы. Если энергия теряется системой кубов, то она приобретается окружающей средой. Энергия никогда не создается и не уничтожается. Поскольку площадь куба-подсказки уменьшилась, визуальная область желтого круга увеличилась. Это символизирует то, как энергия, потерянная системой, приобретается окружающей средой. Воздействие различного окружения и изменения в системе помогают определить увеличение или уменьшение внутренней энергии, тепла и работы.

Таблица 2
Процесс Изменение внутренней энергии Теплопередача тепловой энергии (кв) Работа (w = -PΔV) Пример
q = 0 Адиабатический + 0 + Изолированная система, в которой тепло не проникает и не уходит, подобно пенополистиролу
ΔV = 0 Постоянный объем + + 0 Жесткая система с изолированным давлением, такая как калориметр бомбы
Постоянное давление + или – энтальпия (ΔH) -PΔV Большинство процессов происходят при постоянном внешнем давлении
ΔT = 0 Изотермический 0 + Нет изменений температуры, как в термостате

Пример \ (\ PageIndex {1} \)

Газ в системе имеет постоянное давление.Окружающая среда вокруг системы теряет 62 Дж тепла и выполняет 474 Дж работы с системой. Какова внутренняя энергия системы?

Раствор

Чтобы найти внутреннюю энергию ΔU, мы должны рассмотреть взаимосвязь между системой и окружающей средой. Поскольку Первый закон термодинамики гласит, что энергия не создается и не уничтожается, мы знаем, что все, что теряется в окружающей среде, приобретается системой. Окрестности теряют тепло и работают с системой.Следовательно, q и w положительны в уравнении ΔU = q + w, потому что система накапливает тепло и выполняет работу над собой.

\ [\ begin {align} ΔU & = (62 \, J) + (474 ​​\, J) \\ [4pt] & = 536 \, J \ end {align} \]

Пример \ (\ PageIndex {2} \)

Система имеет постоянный объем (ΔV = 0), а тепло вокруг системы увеличивается на 45 Дж.

  1. Какой знак тепла (q) для системы?
  2. Чему равно ΔU?
  3. Какое значение внутренней энергии системы в Джоулях?

Раствор

Поскольку система имеет постоянный объем (ΔV = 0), член -PΔV = 0 и работа равна нулю.Таким образом, в уравнении ΔU = q + w w = 0 и ΔU = q. Внутренняя энергия равна теплу системы. Окружающее тепло увеличивается, поэтому тепло системы уменьшается, потому что тепло не создается и не разрушается. Таким образом, тепло отводится от системы, делая ее экзотермической и отрицательной. Значение внутренней энергии будет отрицательным значением тепла, поглощаемого окружающей средой.

  1. отрицательный (q <0)
  2. ΔU = q + (-PΔV) = q + 0 = q
  3. ΔU = -45 Дж

Лист термодинамических формул | Список термодинамических формул

1.Внутренняя энергия

Полная энергия составляющих молекул. Он равен сумме внутренней кинетической энергии и потенциальной энергии. Изменение внутренней энергии
dU = (U 2 – U 1 ) = nC v dT
Внутренняя энергия является функцией только состояния, и ее изменение не зависит от пути.

2. Первое начало термодинамики

Выдаваемое тепло равно сумме изменения внутренней энергии и работы, проделанной системой.
δQ = dU + δW

3. Изометрическое или изохорическое изменение

Объем остается постоянным, т.е.
dV = 0
δW = PdV = 0,
δQ = dU = nC v dT.

4. Изобарическое изменение

Давление остается постоянным, т.е.
dP = 0
δQ = dU + δW = nC v dT + PdV

5. Изотермическое изменение

Температура остается постоянной i.е.,
dT = 0
dU = nC v dT = 0, δQ = δW = PdV

6. Адиабатическое изменение

Теплообмен отсутствует, т.е.
δQ = 0
∴ 0 = dU + δW
или
δW = – dU.

7. Бесплатное расширение

Расширение в вакууме. Нет передачи тепла и нет работы, т.е.
δQ = 0, δW = 0 ∴ dU = 0.

8.Уравнение состояния газов

\ (\ frac {P V} {T} \) = Константа
PV = RT (для одного моль газа)
PV = nRT (для n-моль газа).
Изометрическое изменение:
V = постоянная, \ (\ frac {P} {T} \) = постоянная.
Изобарическое изменение:
P = константа, \ (\ frac {V} {T} \) = константа.
Адиабатическое изменение:
Энтропия S = постоянная и ΔQ = 0
PV γ = постоянная
TV γ-1 = постоянная
P 1-γ T γ = постоянная
Pd 1-γ = постоянный.{V_ {2}} \) PdV
Изометрическое изменение:
W = 0
Изобарическое изменение:
W = P (V 2 – V 1 )

Изотермическое изменение

W iso = nRTlog e (V 2 / V 1 )
= nRT 2.303 log e (V 2 / V 1 )

6 = nRT (п. 1 / п. 2 )

Адиабатическое изменение

W ad = \ (\ frac {P_ {1} V_ {1} -P_ {2} V_ {2}} {\ gamma-1} \)
= \ (\ frac {n R } {(\ gamma-1)} \) (T 1 – T 2 )

10.Упругости газов

Изотермическая эластичность E iso = P
Адиабатическая эластичность E adia = γP

Поднимите свои знания предмета на новый уровень, воспользовавшись формулами физики, предоставленными экспертами Onlinecalculator.guru

Что такое термодинамика – получите заметки, книги, формулы и практические вопросы

Если вы потрете руки в течение нескольких секунд, вы обнаружите, что ваши ладони нагреваются, и если вы понаблюдаете за этим явлением с помощью научных очков, вы поймете, что в основном механическая энергия (потирание рук) преобразуется в тепловую энергию (нагретые ладони ).Аналогичным образом возьмем паровой двигатель, в котором тепловая энергия (пар) преобразуется в механическую энергию (движение). Итак, в основном в этой главе вы прочитаете о различных формах энергии и о том, как одна форма энергии преобразуется в другую форму энергии.

Термодинамика Темы

Тепловое равновесие, нулевой закон термодинамики, понятие температуры, тепла, работы и

внутренняя энергия, Первый закон термодинамики, Второй закон термодинамики, обратимые и, необратимые процессы, двигатель Карно и его эффективность.

Обзор термодинамики

В этой главе вы познакомитесь с 4 законами термодинамики, все концепции и вопросы в этой главе вращаются вокруг этих 4 законов. Давайте вкратце разберемся со всеми этими законами.

  • Нулевой закон: Этот закон говорит о концепции температуры, он гласит, что если два тела / системы находятся в тепловом равновесии (без теплообмена) с третьим телом, то все системы находятся в тепловом равновесии друг с другом.

  • Первый закон: изменение внутренней энергии системы равно количеству тепла, добавляемого к системе, за вычетом работы, выполняемой системой. Этот закон также гласит, что энергия всегда сохраняется, ее нельзя ни создать, ни уничтожить, ее можно только передать из одной формы энергии в другую.

  • Второй закон: Проще говоря, он утверждает, что энтропия (возмущение) замкнутой системы всегда увеличивается со временем.Например, если мы возьмем Вселенную как замкнутую систему, ее энтропия всегда увеличивается.

  • Третий закон: Он гласит, что энтропия приближается к постоянному значению, когда температура стремится к абсолютному нулю (0 K).

Слишком сложно понять эти законы? Давайте воспользуемся некоторыми примерами из реальной жизни, чтобы понять эти законы, большинство из вас, должно быть, заметили, что когда вы выходите в очень людных местах, вы начинаете потеть, это явление включает в себя как первый закон, так и второй закон термодинамики, потоотделение происходит до Чтобы охладить наше тело, происходит то, что пот забирает тепло от всего тела, а затем испаряется в атмосфере, и, таким образом, наше тело охлаждается (1-й закон), но в то же время испарившийся пот согревает окружающую среду, приводя к большему теплу и таким образом люди постоянно потеют.Следовательно, вы можете наблюдать, что тепло увеличивается, и это приводит к увеличению энтропии (2-й закон). Вне всякого сомнения, система (людное место) не является закрытой системой, поскольку тепло также передается в окружающую среду, но для нашего понимания мы могли бы принять ее как почти закрытую систему.

Подготовка к работе в сети для JEE Main / NEET

Crack JEE 2021 с программой онлайн-подготовки JEE / NEET

Начать сейчас


Формулы для термодинамики

Как подготовиться к термодинамике

В этой главе вы, в основном, должны сначала понять различные типы процессов, такие как изохорический, изобарный и адиабатический процессы, и вам также следует уделить много внимания диаграммам PV, PT, VT этих процессов. сами диаграммы.Большинство вопросов будет вращаться вокруг концепции Первого закона термодинамики, поэтому постарайтесь ответить на другие вопросы из этой важной темы. Кроме того, многие вопросы будут включать уравнение идеального газа для решения вопросов, настоятельно рекомендуется вам хорошо понять главу Кинетическая теория газов. Без чтения «Кинетической теории газов» вам было бы очень трудно решать вопросы.

Советы по термодинамике
  • Всегда помните о знаковых соглашениях, знаковые соглашения в физике отличаются от химии для термодинамики, потому что в физике мы воспринимаем работу, проделанную системой, как положительную сущность, в то время как в химии мы принимаем работу, проделанную над системой, как положительную.

  • Сохраните список переменных состояния и переменных процесса и помните о различиях между этими двумя примерами переменных: энтропия и внутренняя энергия – это переменные состояния, а работа – это переменная процесса.

  • Разберитесь в различных типах процессов, таких как изобарический, изотермический, адиабатический и изохорный, и научитесь составлять диаграммы этих процессов, такие как P-V, P-T, V-T и т. Д.

  • Продолжайте пересматривать записи и проводите регулярные пробные тесты, чтобы лучше понять эту главу.

Книги по термодинамике

Рекомендация для этой главы – сначала просмотреть книгу NCERT и решить вопросы, после чего вам следует перейти к книге примеров NCERT, чтобы прочувствовать эту главу. Если вы хотите проверить себя на конкурсных экзаменах, вам следует прочитать «Понимание физики» Д.К. Панди или прочитать и попрактиковаться в вопросах из «Концепции физики» Г.К. Верма.

Примечания по физике для инженерных и медицинских экзаменов

12.2 Первый закон термодинамики: тепловая энергия и работа – физика

Задачи обучения раздела

К концу этого раздела вы сможете делать следующее:

  • Опишите, как давление, объем и температура связаны друг с другом и как работают, на основе закона идеального газа
  • Описание работы давление – объем
  • Устно и математически описать первый закон термодинамики
  • Решение задач, связанных с первым законом термодинамики

Поддержка учителей

Поддержка учителей

Цели обучения в этом разделе помогут вашим ученикам овладеть следующими стандартами:

  • (6) Научные концепции.Учащийся знает, что в физической системе происходят изменения, и применяет законы сохранения энергии и количества движения. Ожидается, что студент:
    • (Г) анализировать и объяснять повседневные примеры, иллюстрирующие законы термодинамики, включая закон сохранения энергии и закон энтропии.

Раздел Ключевые термины

Постоянная Больцмана первый закон термодинамики Закон об идеальном газе внутренняя энергия давление

Давление, объем, температура и закон идеального газа

Поддержка учителей

Поддержка учителей

[BL] [OL] [AL] Просмотрите понятие силы.

[ПР] Спросите студентов, с какой силой нужно забить гвоздь в стену. Достигли бы они того же результата, если бы гвоздь был тупым, а не заостренным? Почему или почему нет?

Прежде чем рассматривать первый закон термодинамики, сначала важно понять взаимосвязь между давлением, объемом и температурой. Давление, P , определяется как

, где F – сила, приложенная к области, A , которая перпендикулярна силе.

В зависимости от области воздействия заданная сила может иметь существенно разный эффект, как показано на Рисунке 12.3.

Рис. 12.3 (a) Хотя человек, которого тыкают пальцем, может раздражаться, сила не имеет длительного эффекта. (б) Напротив, та же сила, приложенная к области размером с острый конец иглы, достаточно велика, чтобы повредить кожу.

Единицей измерения давления в системе СИ является паскаль , где 1 Па = 1 Н / м2, 1 Па = 1 Н / м2.

Давление определяется для всех состояний вещества, но особенно важно при обсуждении жидкостей (например, воздуха). Вы, наверное, слышали, что слово давление используется по отношению к крови (высокое или низкое кровяное давление) и по отношению к погоде (погодные системы с высоким и низким давлением). Это только два из многих примеров давления в жидкости.

Соотношение между давлением, объемом и температурой для идеального газа определяется законом идеального газа.Газ считается идеальным при низком давлении и довольно высокой температуре, и силами между составляющими его частицами можно пренебречь. Согласно закону идеального газа,

, где P – давление газа, V – объем, который он занимает, N – количество частиц (атомов или молекул) в газе, а T – его абсолютная температура. Константа k называется постоянной Больцмана и имеет значение k = 1,38 × 10–23 Дж / К, k = 1,38 × 10–23 Дж / К. Для целей этой главы мы не будем вдаваться в вычисления с использованием идеального газовое право.Вместо этого для нас важно заметить из уравнения, что для данной массы газа верно следующее:

  • Когда объем постоянный, давление прямо пропорционально температуре.
  • Когда температура постоянна, давление обратно пропорционально объему.
  • Когда давление постоянно, объем прямо пропорционален температуре.

Этот последний пункт описывает тепловое расширение – изменение размера или объема данной массы в зависимости от температуры.Что является основной причиной теплового расширения? Повышение температуры означает увеличение кинетической энергии отдельных атомов. Газы особенно подвержены тепловому расширению, хотя жидкости расширяются в меньшей степени при аналогичном повышении температуры, и даже твердые тела имеют незначительное расширение при более высоких температурах. Вот почему железнодорожные пути и мосты имеют компенсаторы, которые позволяют им свободно расширяться и сжиматься при изменении температуры.

Поддержка учителей

Поддержка учителей

[BL] [OL] [AL] Сообщите учащимся, что закон идеального газа строго верен только для газов, которые идеальны .Настоящие газы в чем-то отличаются от них. Например, предполагается, что в идеальном газе нет межмолекулярных сил, все столкновения между молекулами совершенно упругие и т. Д. Однако закон идеального газа полезен в расчетах, потому что при стандартных условиях температуры и давления многие реальные газы, такие как кислород, азот, водород и благородные газы, демонстрируют качественное поведение, очень близкое к поведению идеального газа.

Чтобы получить некоторое представление о том, как давление, температура и объем газа связаны друг с другом, рассмотрим, что происходит, когда вы закачиваете воздух в спущенную шину.Сначала объем шины увеличивается прямо пропорционально количеству впрыскиваемого воздуха без значительного увеличения давления в шине. Когда шина расширилась почти до своего полного размера, стенки ограничивают объемное расширение. Если вы продолжите нагнетать воздух в шину (которая теперь имеет почти постоянный объем), давление возрастет с увеличением температуры (см. Рисунок 12.4).

Рис. 12.4 (a) Когда воздух нагнетается в спущенную шину, его объем сначала увеличивается без значительного увеличения давления.(b) Когда шина заполнена до определенной точки, стенки шины сопротивляются дальнейшему расширению, и давление увеличивается по мере добавления воздуха. (c) После того, как шина полностью накачана, ее давление увеличивается с температурой.

Работа «давление – объем»

Работа «давление – объем» – это работа, которая выполняется за счет сжатия или расширения жидкости. Когда происходит изменение объема, а внешнее давление остается постоянным, происходит работа давления и объема. Во время сжатия уменьшение объема увеличивает внутреннее давление в системе, поскольку работа выполняется на системе.Во время расширения (рис. 12.5) увеличение объема снижает внутреннее давление в системе, поскольку система выполняет работу .

Рис. 12.5 Расширение газа требует передачи энергии для поддержания постоянного давления. Поскольку давление постоянно, проделанная работа равна PΔVPΔV. .

Напомним, что формула работы: W = Fd.W = Fd. Мы можем изменить определение давления, P = FA, P = FA, чтобы получить выражение для силы в терминах давления.

Подставляя это выражение для силы в определение работы, получаем

Поскольку площадь, умноженная на смещение, является изменением объема, W = PΔVW = PΔV , математическое выражение для работы давление – объем равно

Так же, как мы говорим, что работа – это сила, действующая на расстоянии, для жидкостей мы можем сказать, что работа – это давление, действующее через изменение объема.Для работы давление – объем давление аналогично силе, а объем аналогичен расстоянию в традиционном определении работы.

Watch Physics

Работа с расширением

В этом видео описывается работа с расширением (или работа давление – объем). Sal объединяет уравнения W = PΔVW = PΔV и ΔU = Q − WΔU = Q − W чтобы получить ΔU = Q − PΔVΔU = Q − PΔV .

Проверка захвата

Если объем системы увеличивается, а давление остается постоянным, значение работы, проделанной системой W , положительное или отрицательное? Будет ли это увеличивать или уменьшать внутреннюю энергию системы?

  1. Положительный; внутренняя энергия уменьшится
  2. Положительный; внутренняя энергия увеличится
  3. отрицательный; внутренняя энергия уменьшится
  4. отрицательный; внутренняя энергия увеличится

Первый закон термодинамики

Поддержка учителя

Поддержка учителя

[BL] Проверьте теплопередачу.Когда и как происходит передача тепла между двумя телами? Что происходит, когда энергия передается в систему или из нее под действием тепла?

Тепло ( Q ) и работа ( W ) – это два способа добавления или удаления энергии из системы. Процессы очень разные. Тепло вызывается разницей температур, а работа связана с силой, действующей на расстоянии. Тем не менее тепло и работа могут дать одинаковые результаты. Например, оба могут вызвать повышение температуры. Тепло передает энергию системе, например, когда солнце нагревает воздух в велосипедной шине и повышает температуру воздуха.Точно так же можно работать с системой, например, когда велосипедист нагнетает воздух в шину. Как только произошло повышение температуры, невозможно сказать, было ли оно вызвано жарой или работой. Тепло и работа – это энергия в пути, и ни одна из них не хранится как таковая в системе. Однако оба могут изменять внутреннюю энергию U системы.

Внутренняя энергия – это сумма кинетической и потенциальной энергий атомов и молекул системы. Его можно разделить на множество подкатегорий, таких как тепловая и химическая энергия, и зависит только от состояния системы (то есть P , V и T ), а не от того, как энергия входит или выходит. система.

Чтобы понять взаимосвязь между теплотой, работой и внутренней энергией, мы используем первый закон термодинамики. Первый закон термодинамики применяет принцип сохранения энергии к системам, в которых тепло и работа являются методами передачи энергии в системы и из них. Его также можно использовать для описания того, как энергия, передаваемая теплом, преобразуется и снова передается работой.

Поддержка учителей

Поддержка учителей

[OL] [AL] Попросите учащихся привести примеры процессов, при которых энергия преобразуется из одной формы в другую.Проанализируйте, происходит ли передача энергии за счет тепла в каждом конкретном случае.

Советы для успеха

Напомним, что принцип сохранения энергии гласит, что энергия не может быть создана или уничтожена, но она может быть изменена из одной формы в другую.

Первый закон термодинамики гласит, что изменение внутренней энергии закрытой системы равно чистой теплопередаче в системе за вычетом чистой работы, выполненной системой. В форме уравнения первый закон термодинамики равен

.

Здесь ΔUΔU – изменение внутренней энергии , U системы.Как показано на рис. 12.6, Q – это чистое тепло , переданное в систему , то есть Q – это сумма всех теплопередач в систему и из нее. W – это чистая работа, выполненная системой , то есть W – это сумма всей работы, выполненной в системе или ею. По соглашению, если значение Q положительное, то в системе имеется чистая теплопередача; если W положительный, значит, система выполняет чистую работу. Таким образом, положительный Q добавляет энергию в систему за счет тепла, а положительный W забирает энергию из системы за счет работы.Обратите внимание, что если тепло передает системе больше энергии, чем та, которую производит работа, разница сохраняется как внутренняя энергия.

Рис. 12.6 Первый закон термодинамики – это принцип сохранения энергии , установленный для системы, где тепло и работа являются методами передачи энергии в систему и из нее. Q представляет собой чистую теплопередачу – это сумма всех передач тепла в систему и из нее. Q является положительным для чистой передачи тепла в систему.WoutWout – это работа, выполненная системой , а WinWin – это работа, выполненная в системе . Вт – это общее количество работ, выполненных в системе, или . W является положительным, когда выполняет больше работы системой, чем ее. Изменение внутренней энергии системы ΔUΔU связано с теплом и работой по первому закону термодинамики: ΔU = Q − W.ΔU = Q − W.

Отсюда также следует, что отрицательное значение Q указывает на то, что энергия передается от от системы за счет тепла и, таким образом, уменьшает внутреннюю энергию системы, тогда как отрицательное значение Вт, – это работа, совершенная на системе, что увеличивает внутреннюю энергию.

Watch Physics

Первый закон термодинамики / внутренняя энергия

Это видео объясняет первый закон термодинамики, сохранения энергии и внутренней энергии. Он рассматривает пример преобразования энергии между кинетической энергией, потенциальной энергией и теплопередачей из-за сопротивления воздуха.

Проверка захвата

Смотреть Физика: Первый закон термодинамики / Внутренняя энергия. Это видео вводит и объясняет первый закон термодинамики и концепцию внутренней энергии.

Рассмотрим пример подбрасывания мяча при сопротивлении воздуха. Что, по вашему мнению, произойдет с конечной скоростью и конечной кинетической энергией мяча по мере увеличения сопротивления воздуха? Почему?

  1. Оба уменьшатся. Энергия передается воздуху за счет тепла из-за сопротивления воздуха.
  2. Оба увеличатся. Энергия передается от воздуха к мячу за счет сопротивления воздуха.
  3. Конечная скорость увеличится, но конечная кинетическая энергия уменьшится.Энергия передается от мяча к воздуху через сопротивление воздуха.
  4. Конечная скорость уменьшится, но конечная кинетическая энергия увеличится. Энергия передается от воздуха к мячу посредством тепла через сопротивление воздуха.

Поддержка учителей

Поддержка учителей

[BL] [OL] Продемонстрируйте, что уравнение можно переформулировать как Q = ΔU + W.Q = ΔU + W. Это показывает, что любая энергия, добавляемая к системе за счет тепла, либо преобразуется в работу, либо сохраняется как внутренняя энергия.

Watch Physics

Подробнее о внутренней энергии

В этом видео более подробно рассказывается о внутренней энергии и о том, как использовать уравнение ΔU = Q − W. ΔU = Q − W. Обратите внимание, что Sal использует уравнение ΔU = Q + WΔU = Q + W , где W – это работа, выполненная на в системе, тогда как мы используем W для представления работы, выполненной системой.

Проверка захвата

Если 5 \, \ text {J} уносятся теплом из системы, и система выполняет 5 \, \ text {J} работы, каково изменение внутренней энергии системы?

  1. {-10} \, \ text {J}
  2. 0 \, \ text {J}
  3. 10 \, \ text {J}
  4. 25 \, \ text {J}

Ссылки на физику

Биология: биологическая термодинамика

Мы часто думаем о термодинамике как о полезной для изобретения или тестирования оборудования, такого как двигатели или паровые турбины.Однако термодинамика также применима к живым системам, таким как наши собственные тела. Это составляет основу биологической термодинамики (рис. 12.7).

Рис. 12.7 (a) Первый закон термодинамики применим к метаболизму. Тепло, передаваемое из тела (Q), и работа, выполняемая телом (W), удаляют внутреннюю энергию, тогда как прием пищи заменяет ее. (Прием пищи можно рассматривать как работу, выполняемую телом.) (Б) Растения преобразуют часть лучистой энергии солнечного света в запасенную химическую энергию – процесс, называемый фотосинтез .

Сама жизнь зависит от биологической передачи энергии. Посредством фотосинтеза растения поглощают солнечную энергию и используют эту энергию для преобразования углекислого газа и воды в глюкозу и кислород. Фотосинтез принимает одну форму энергии – свет – и преобразует ее в другую форму – химическую потенциальную энергию (глюкозу и другие углеводы).

Метаболизм человека – это преобразование пищи в энергию, выделяемую теплом, работу, выполняемую клетками организма, и накопленный жир.Метаболизм – интересный пример действия первого закона термодинамики. Прием пищи увеличивает внутреннюю энергию тела за счет добавления химической потенциальной энергии; это неромантичный взгляд на хороший буррито.

Организм усваивает всю пищу, которую мы потребляем. По сути, метаболизм – это процесс окисления, при котором высвобождается химическая потенциальная энергия пищи. Это означает, что питание осуществляется в форме работы. Упражнения помогают вам похудеть, поскольку они обеспечивают передачу энергии от вашего тела как за счет тепла, так и за счет работы, а также повышают уровень метаболизма, даже когда вы находитесь в состоянии покоя.

Биологическая термодинамика также включает изучение трансдукции между клетками и живыми организмами. Трансдукция – это процесс, при котором генетический материал – ДНК – передается от одной клетки к другой. Это часто происходит во время вирусной инфекции (например, гриппа), и именно так вирус распространяется, а именно путем передачи своего генетического материала все большему количеству ранее здоровых клеток. Как только достаточное количество клеток заражается, вы начинаете ощущать воздействие вируса (симптомы гриппа – мышечная слабость, кашель и заложенность носа).

Энергия передается вместе с генетическим материалом и, таким образом, подчиняется первому закону термодинамики. Энергия передается – а не создается и не уничтожается – в процессе. Когда с элементом выполняется работа или тепло передает энергию ячейке, внутренняя энергия ячейки увеличивается. Когда клетка работает или теряет тепло, ее внутренняя энергия уменьшается. Если количество работы, выполняемой ячейкой, такое же, как количество энергии, передаваемой теплом, или количество работы, выполняемой ячейкой, соответствует количеству энергии, передаваемой теплом, чистого изменения внутренней энергии не будет. .

Проверка захвата

Исходя из того, что вы знаете о теплопередаче и первом законе термодинамики, нужно ли вам есть больше или меньше, чтобы поддерживать постоянный вес в холодную погоду? Объяснить, почему.

    еще
  1. ; поскольку в холодную погоду организм теряет больше энергии, потребность в еде увеличивается, чтобы поддерживать постоянный вес
  2. Еще
  3. ; употребление большего количества пищи означает накопление большего количества жира, что защитит организм от холодной погоды и уменьшит потерю энергии
  4. На
  5. меньше; поскольку в холодную погоду организм теряет меньше энергии, потребность в еде уменьшается, чтобы поддерживать постоянный вес
  6. На
  7. меньше; употребление меньшего количества пищи означает накопление меньшего количества жира, поэтому для сжигания жира потребуется меньше энергии, и в результате вес останется постоянным

Решение задач, связанных с первым законом термодинамики

Рабочий пример

Расчет изменения внутренней энергии

Предположим, 40.00 Дж энергии передается системе с помощью тепла, в то время как система выполняет 10,00 Дж работы. Позже тепло передает 25,00 Дж из системы, в то время как 4,00 Дж приходится на работу в системе. Каково чистое изменение внутренней энергии системы?

Стратегия

Сначала необходимо рассчитать чистое тепло и чистую работу. Затем, используя первый закон термодинамики, ΔU = Q − W, ΔU = Q − W, найдите изменение внутренней энергии.

Решение

Чистое тепло – это передача тепла в систему за вычетом передачи тепла из системы, или

Q = 40.00 Дж – 25,00 Дж = 15,00 Дж. Q = 40,00 Дж – 25,00 Дж = 15,00 Дж.

12,7

Общая работа – это работа, выполненная системой за вычетом работы, выполненной в системе, или

W = 10,00 Дж – 4,00 Дж = 6,00 Дж. W = 10,00 Дж – 4,00 Дж = 6,00 Дж.

12,8

Изменение внутренней энергии определяется первым законом термодинамики.

ΔU = Q − W = 15,00 Дж − 6,00 Дж = 9,00 Дж ΔU = Q − W = 15,00 Дж − 6,00 Дж = 9,00 Дж

12,9

Обсуждение

Другой способ решить эту проблему – найти изменение внутренней энергии для каждого из двух шагов отдельно, а затем сложить два изменения, чтобы получить общее изменение внутренней энергии.Такой подход будет выглядеть следующим образом:

Для 40,00 Дж тепла на входе и 10,00 Дж на тренировке изменение внутренней энергии составляет

ΔU1 = Q1 − W1 = 40,00 Дж − 10,00 Дж = 30,00 Дж. ΔU1 = Q1 − W1 = 40,00 Дж − 10,00 Дж = 30,00 Дж.

12,10

При 25,00 Дж отвода тепла и 4,00 Дж работы на входе изменение внутренняя энергия

ΔU2 = Q2 − W2 = −25,00 Дж – (- 4,00 Дж) = – 21,00 Дж. ΔU2 = Q2 − W2 = −25,00 Дж – (- 4,00 Дж) = – 21,00 Дж.

12,11

Общее изменение составляет

ΔU = ΔU1 + ΔU2 = 30,00 Дж + (- 21,00 Дж) = 9,00 Дж. ΔU = ΔU1 + ΔU2 = 30.00 Дж + (- 21,00 Дж) = 9,00 Дж.

12,12

Независимо от того, смотрите ли вы на процесс в целом или разбиваете его на этапы, изменение внутренней энергии одинаково.

Поддержка учителей

Поддержка учителей

[BL] Убедитесь, что учащиеся четко понимают использование отрицательных знаков для передачи тепла и работы. Работа, выполненная на в изолированной системе, означает увеличение объема, поэтому W является положительным, а ΔUΔU уменьшается или отрицательным.

Рабочий пример

Расчет изменения внутренней энергии: одно и то же изменение в
U производится двумя разными процессами

Каково изменение внутренней энергии системы, когда всего 150.00 Дж передается за счет тепла от системы, а 159,00 Дж приходится на работу в системе?

Стратегия

Полезное тепло и работа уже заданы, поэтому просто используйте эти значения в уравнении ΔU = Q − W. ΔU = Q − W.

Решение

Здесь чистое тепло и общая работа даны непосредственно как Q = -150,00 Дж и W = -159,00 Дж, Q = -150,00 Дж и W = -159,00 Дж, так что

ΔU = Q − W = −150,00 Дж – (- 159,00 Дж) = 9,00 Дж. ΔU = Q − W = −150,00 Дж – (- 159,00 Дж) = 9,00 Дж.

12,13

Обсуждение

Рисунок 12.8 Два разных процесса производят одно и то же изменение в системе. (а) Всего 15,00 Дж теплопередачи происходит в системе, в то время как работа забирает в общей сложности 6,00 Дж. Изменение внутренней энергии ΔU = Q – W = 9,00 Дж. (Б) Теплообмен удаляет 150,00 Дж из системы, в то время как работа добавляет в нее 159,00 Дж, что дает увеличение на 9,00 Дж во внутренней энергии. Если система запускается в том же состоянии в (a) и (b), она закончится в одном и том же конечном состоянии в любом случае – его конечное состояние связано с внутренней энергией, а не как эта энергия была получена.

Совершенно другой процесс во втором отработанном примере дает такое же изменение внутренней энергии на 9,00 Дж, что и в первом отработанном примере. Обратите внимание, что изменение в системе в обеих частях связано с ΔUΔU, а не с отдельными задействованными Q или W . В обеих задачах система оказывается в том же состоянии . Обратите внимание, что, как обычно, на Рисунке 12.8 выше, WoutWout выполняется работа по системе, а WinWin проделана работа по системе.

Практические задачи

3.

Сколько работы совершает газ под давлением 20 Па, увеличивающийся в объеме на 3,0 м 3 ?

  1. –0,15 Дж
  2. 6,7 Дж
  3. –23 Дж
  4. 60 Дж
4.

Что такое чистое тепло, выходящее из системы, когда 25 \, \ text {J} передается за счет тепла в систему, а 45 \, \ text {J} передается из нее?

  1. {-70} \, \ text {J}
  2. {-20} \, \ text {J}
  3. 20 \, \ text {J}
  4. 70 \, \ text {J}

Проверьте свое понимание

Поддержка учителей

Поддержка учителей

Используйте эти вопросы, чтобы оценить достижения учащимися учебных целей раздела.Если учащиеся борются с какой-то конкретной целью, эти вопросы помогут определить, какие учащиеся, и направить их к соответствующему содержанию.

5.

Что такое давление?

  1. Давление – это сила, разделенная на длину.
  2. Давление – это сила, разделенная на площадь.
  3. Давление – это сила, разделенная на объем.
  4. Давление делится на силу, деленную на массу.
6.

Что такое единица СИ для давления?

  1. паскаль, или Н / м 3
  2. кулон
  3. ньютон
  4. паскаль, или Н / м 2
7.

Что такое работа «давление-объем»?

  1. Это работа, которая выполняется за счет сжатия или расширения жидкости.
  2. Это работа, которая выполняется силой над объектом для создания определенного смещения.
  3. Это работа, которую совершают поверхностные молекулы жидкости.
  4. Это работа, которую совершают высокоэнергетические молекулы жидкости.
8.

Когда считается, что работа по давлению-объему выполняется в системе?

  1. Когда увеличивается как объем, так и внутреннее давление.
  2. Когда наблюдается уменьшение как объема, так и внутреннего давления.
  3. При уменьшении объема и повышении внутреннего давления.
  4. Когда происходит увеличение объема и уменьшение внутреннего давления.
9.

Каким образом можно добавить или удалить энергию из системы?

  1. Передача энергии посредством тепла – это единственный способ добавить или удалить энергию из системы.
  2. Выполнение работы по сжатию – единственный способ добавить или удалить энергию из системы.
  3. Выполнение работ по расширению – единственный способ добавить или убрать энергию из системы.
  4. Передача энергии посредством тепла или выполнения работы – это способы добавления энергии в систему или ее удаления из системы.
10.

Что такое внутренняя энергия?

  1. Это сумма кинетических энергий атомов и молекул системы.
  2. Это сумма потенциальных энергий атомов и молекул системы.
  3. Это сумма кинетической и потенциальной энергий атомов и молекул системы.
  4. Это разница между величинами кинетической и потенциальной энергий атомов и молекул системы.
.

Оставить комментарий